Вы находитесь на странице: 1из 27

1

INSURANCE-CONCEALMENT
Republic of the Philippines
SUPREME COURT
Manila
EN BANC
G.R. No. L-24899 March 19, 1928
BERNARDO ARGENTE, plaintiff-appellant,
vs.
WEST COAST LIFE INSURANCE CO., defendant-appellee.
Abad Santos, Camus, Delgado & Recto for appellant.
Gibbs & McDonough and Roman Ozaeta for appellee.
MALCOLM, J.:
This is an action upon a joint life insurance policy for P15,000 issued by the defendant, the
West Coast Life Insurance Co., on May 15, 1925, in favor of the plaintiff, Bernardo Argente, and
his wife, Vicenta de Ocampo, the latter having died on November 18, 1925. Fraud in obtaining
the policy was pleaded by way of special defense. On the issue thus suggested, the court
adopted the theory of the defendant, and held the insurance policy null and void, with the
result that the complaint was dismissed, with costs.
On February 9, 1925, Bernardo Argente signed an application for joint insurance with his wife
in the sum of P2,000. The wife, Vicenta de Ocampo, signed a like application for the same
policy. Both applications, with the exception of the names and the signatures of the applicants,
were written by Jose Geronimo del Rosario, an agent for the West Coast Life Insurance Co. But
all the information contained in the applications was furnished the agent by Bernardo Argente.
Pursuant to his application, Bernardo Argente was examined by Dr. Cesareo Sta. Ana, a medical
examiner for the West Coast Life Insurance Co., on February 10, 1925, in the office of the
Customs House. The result of such examination was recorded in the Medical Examiner's
Report, and with the exception of the signature of Bernardo Argente, was in the hand-writing
of Doctor Sta. Ana. But the information or answers to the questions contained on the face of
the Medical Examiner's Report were furnished the doctor by the applicant, Bernardo Argente.
Pursuant to her application, Vicenta de Ocampo, wife of the plaintiff, was examined by Dr.
Cesareo Sta. Ana on February 10, 1925, at her residence in Manila. The result of the medical
examination, including among other things, the answers given by Vicenta de Ocampo to the
questions propounded to her by the physician, appears in the Medical Examiner's Report.
On May 9, 1925, Bernardo Argente and his wife submitted to the West Coast Life Insurance Co.
an amended application for insurance, increasing the amount thereof to P15,000, and asked
that the policy be dated May 15, 1925. The amended application was accompanied by the
documents entitled "Short Form Medical Report." In both of these documents appear certain
questions and answers.
A temporary policy for P15,000 was issued to Bernardo Argente and his wife as of May 15, but
it was not delivered to Bernardo Argente until July 2, 1925, when the first quarterly premium
on the policy was paid. In view of the fact that more than thirty days had elapsed since the
applicants were examined by the company's physician, each of them was required to file a
certificate of health before the policy was delivered to them.
On November 18, 1925, Vicenta de Ocampo died of cerebral apoplexy. Thereafter Bernardo
Argente presented a claim in due form to the West Coast Life Insurance Co. for the payment of
the sum of P15,000 the amount of the joint life Insurance policy. Following investigation
conducted by the Manager of the Manila office of the insurance company, it was apparently
disclosed that the answers given by the insured in their medical examinations with regard to
their health and previous illness and medical attendance were untrue. For that reason, the
West Coast Life Insurance Co. refused to pay the claim of Bernardo Argente, and on May 25,
1926, wrote him to the effect that the claim was rejected because the insurance was obtained
through fraud and misrepresentation.
It is admitted that it appears in the Medical Examiner's Report that Bernardo Argente, in
response to the question asked by the medical examiner, "Have you ever consulted a physician
for, or have you ever suffered from any ailment or disease of, the brain or nervous system?"
answered "No." To the question, "Have you consulted a physician for any ailment or disease
not included in your above answer," answered "Yes. Nature of Ailment, Disease or Injury.
Scabies, Number of attacks 1, Date 1911. Duration 1 month, Severity Fair, results and, if within
five years, name and address of every physician consulted. Dr. P. Guazon. Cured. Dr. Guazon is
dead now." And to the question, "What physician or physicians, if any, not named above, have
you consulted or been treated by, within the last five years and for what illness or ailment? (If
none, so state)" answered "No." It is, however, not disputed that on January 10, 11, and 13,
1923, Bernardo Argente was confined in the Philippine General Hospital where he was treated
by Dr. Agerico B. M. Sison for cerebral congestion and Bell's Palsy.
It is further admitted that it appears in the Medical Examiner's Report that Vicenta de Ocampo,
in response to the question asked by the medical examiner, "How frequently, if at all, and in
what quantity do you use beer, wine, spirits or other intoxicants?" answered "Beer only in
small quantities occasionally." To the question, "Have you ever consulted a physician for or
have you ever suffered from any ailment or disease of the brain or nervous system?" answered
"No." To the question, "What physician or physicians, if any, not named above, have you
consulted or been treated by, within the last five years and for what illness or ailment? (If
none, so state)" answered "None." And to the question, "Are you in good health as far as you
know and believe?" answered "Yes." It is, however, not disputed that Vicenta de Ocampo was
taken by a patrolman, at the request of her husband, Bernardo Argente, on May 19, 1924, to
the Meisic police station, and from there was transferred to the San Lazaro Hospital. In San
Lazaro Hospital, her case was diagnosed by the admitting physician as "alcoholism," but later
2

Doctor Domingo made a diagnosis of probable "manic-depressive psychosis," and still, later in
Mary Chiles Hospital, made a final diagnosis of "phycho-neurosis."
The plaintiff, Bernardo Argente, while readily conceding most of the facts herein narrated, yet
alleges that both he and his wife revealed to the company's physician. Doctor Sta. Ana, all the
facts concerning the previous illnesses and medical attendance, but that Doctor Sta. Ana,
presumably acting in collusion, with the insurance agent, Jose Geronimo del Rosario, failed to
record them in the medical reports. The evidence on these points consists of the testimony of
the plaintiff and his subordinate clerk, Apolonio Espiritu, on the one hand, and of the
testimony of Doctor Sta. Ana and Jose Geronimo del Rosario on the other. On the question of
fact thus raised, the trial judge found with the insurance company. In so doing, we believe that
His Honor gave proper inclination to the weight of the proof. There appears no motive
whatever on the part of Doctor Sta. Ana to falsify the Medical Examiner's Reports and thereby
not only jeopardize his career as a physician, but also gravely implicate himself criminally.
What has heretofore been stated in this decision is gleaned to a great extent the carefully
prepared decision of the trial judge, the Honorable George R. Harvey. The court found from the
evidence that the representations made by Bernardo Argente and his wife in their applications
to the defendant for life insurance were false with respect to their estate of health during the
period of five years preceding the date of such applications, and that they knew the
representations made by them in their applications were false. The court further found from
the evidence that the answers given by Bernardo Argente and his wife at the time of the
medical examination by Doctor Sta. Ana were false with respect to the condition of their health
at that time and for a period of several years prior thereto. Based on these findings which must
here be accepted since the stenographic transcript is incomplete, the question arises as to the
estate of the law in relation thereto.
One ground for the rescission of a contract of insurance under the Insurance Act is "a
concealment," which in section 25 is defined as "A neglect to communicate that which a party
knows and ought to communicate." Appellant argues that the alleged concealment was
immaterial and insufficient to avoid the policy. We cannot agree. In an action on a life
insurance policy where the evidence conclusively shows that the answers to questions
concerning diseases were untrue, the truth of falsity of the answers become the determining
factor. In the policy was procured by fraudulent representations, the contract of insurance
apparently set forth therein was never legally existent. It can fairly be assumed that had the
true facts been disclosed by the assured, the insurance would never have been granted.
In Joyce, The Law of Insurance, second edition, volume 3, Chapter LV, is found the following:
Concealment exists where the assured has knowledge of a fact material to the risk,
and honesty, good faith, and fair dealing requires that he should communicate it to
the assured, but he designated and intentionally with holds the same.
Another rule is that if the assured undertakes to state all the circumstances affecting
the risk, a full and fair statement of all is required.
It is also held that the concealment must, in the absence of inquiries, be not only
material, but fraudulent, or the fact must have been intentionally withheld; so it is
held under English law that if no inquiries are made and no fraud or design to conceal
enters into the concealment the contract is not avoided. And it is determined that
even though silence may constitute misrepresentation or concealment it is not itself
necessarily so as it is a question of fact. Nor is there a concealment justifying a
forfeiture where the fact of insanity is not disclosed no questions being asked
concerning the same. . . .
But it would seem that if a material fact is actually known to the assured, its
concealment must of itself necessarily be a fraud, and if the fact is one which the
assured ought to know, or is presumed to know, the presumption of knowledge
ought to place the assured in the same position as in the former case with relation to
material facts; and if the jury in such cases find the fact material, and one tending to
increase the risk, it is difficult to see how the inference of a fraudulent intent or
intentional concealment can be avoided. And it is declared that if a material fact
concealed by assured it is equivalent to a false representation that it does not exist
and that the essentials are the truth of the representations whether they were
intended to mislead and did insurer accept them as true and act upon them to his
prejudice. So it is decided that under a stipulation voiding the policy for concealment
or misrepresentation of any material fact or if his interest is not truly stated or is
either than the sole and unconditional ownership the facts are unimportant that
insured did not intend to deceive or withhold information as to encumbrances even
though no questions were asked. And if insured while being examined for life
insurance and knowing that she had heart disease, falsely stated that she was in
good health, and though she could not read the application, it was explained to her
and the questions asked through an interpreter, and the application like the policy
contained and provision that no liability should be incurred unless the policy was
delivered while the insured was in good health, the court properly directed a verdict
for the insurer, though a witness who was present at the examination testified that
the insured was not asked whether she had heart disease.
x x x x x x x x x
The basis of the rule vitiating the contract in case of concealment is that it misleads
or deceives the insurer into accepting the risk, or accepting it at the rate of premium
agreed upon. The insurer, relying upon the belief that the assured will disclose every
material within his actual or presumed knowledge, is misled into a belief that the
circumstance withheld does not exist, and he is thereby induced to estimate the risk
upon a false basis that it does not exist. The principal question, therefore, must be,
Was the assurer misled or deceived into entering a contract obligation or in fixing the
premium of insurance by a withholding of material information of facts within the
assured's knowledge or presumed knowledge?
It therefore follows that the assurer in assuming a risk is entitled to know every
material fact of which the assured has exclusive or peculiar knowledge, as well as all
material facts which directly tend to increase the hazard or risk which are known by
3

the assured, or which ought to be or are presumed to be known by him. And a
concealment of such facts vitiates the policy. "It does not seem to be necessary . . .
that the . . . suppression of the truth should have been willful." If it were but an
inadvertent omission, yet if it were material to the risk and such as the plaintiff
should have known to be so, it would render the policy void. But it is held that if
untrue or false answers are given in response to inquiries and they relate to material
facts the policy is avoided without regard to the knowledge or fraud of assured,
although under the statute statements are representations which must be fraudulent
to avoid the policy. So under certain codes the important inquiries are whether the
concealment was willful and related to a matter material to the risk.
x x x x x x x x x
If the assured has exclusive knowledge of material facts, he should fully and fairly
disclose the same, whether he believes them material or not. But notwithstanding
this general rule it will not infrequently happen, especially in life risks, that the
assured may have a knowledge actual or presumed of material facts, and yet
entertain an honest belief that they are not material. . . . The determination of the
point whether there has or has not been a material concealment must rest largely in
all cases upon the form of the questions propounded and the exact terms of the
contract. Thus, where in addition to specifically named diseases the insured was
asked whether he had any sickness within ten years, to which he answered "No," and
it was proven that within that period he had a slight of pharyngitis, it was held a
question properly for the jury whether such an inflammation of the throat was a
"sickness" within the intent of the inquiry, and the court remarked on the appeal
decision that if it could be held as a matter of law that the policy was thereby
avoided, then it was a mere device on the part of insurance companies to obtain
money without rendering themselves liable under the policy. . . .
. . . The question should be left to the jury whether the assured truly represented the
state of his health so as not mislead or deceive the insurer; and if he did not deal a
good faith with insurer in that matter, that the inquiry should be made, Did he know
the state of his health so as to be able to furnish a proper answer to such questions
as are propounded? A Massachusetts case, if construed as it is frequently cited,
would be opposed to the above conclusion; but, on the contrary, it sustains it, for the
reason that symptoms of consumption had so far developed themselves within a few
months prior to effecting the insurance as to induce a reasonable belief that the
applicant had that fatal disease, and we should further construe this case as
establishing the rule that such a matter cannot rest alone upon the assured's belief
irrespective of what is a reasonable belief, but that it ought to be judged by the
criterion whether the belief is one fairly warranted by the circumstances. A case in
Indiana, however, holds that if the assured has some affection or ailment of one or
more of the organs inquired about so well-defined and marked as to materially
derange for a time the functions of such organ, as in the case of Bright's disease, the
policy will be avoided by a nondisclosure, irrespective of the fact whether the
assured knew of such ailment or not. . . .
Lastly, appellant contends that even if the insurance company had a right to rescind the
contract, such right cannot now be enforced in view of the provisions of section 47 of the
Insurance Act providing "Whenever a right to rescind a contract of insurance is given to their
insurer by provision of this chapter, such right must be exercised previous to the
commencement of an action on the contract." This section was derived from section 2583 of
the California Civil Code, but in contrast thereto, makes use of the imperative "must" instead of
the permissive "may." Nevertheless, there are two answers to the problem as propounded.
The first is that the California law as construed by the code examiners, at whose
recommendation it was adopted, conceded that "A failure to exercise the right (of rescission),
cannot, of course, prejudice any defense to the action which the concealment may furnish."
(Codes of California annotated; Tan Chay Heng vs. West Coast Life Insurance Company [1927],
p. 80,ante.) The second answer is that the insurance company more than one month previous
to the commencement of the present action wrote the plaintiff and informed him that the
insurance contract was void because it had been procured through fraudulent representations,
and offered to refund to the plaintiff the premium which the latter had paid upon the return of
the policy for cancellation. As held in California as to a fire insurance policy, where any of the
material representations are false, the insurer's tender of the premium and notice that the
policy is canceled, before the commencement of suit thereon, operate to rescind the contract
of insurance. (Rankin vs.Amazon Insurance Co. [1891], 89 Cal., 203.)
We are content to rest our judgment on the findings of the trial court, and on the law
governing those facts, with the result that the various assignments of error are found to be
without persuasive merit.
Judgment affirmed, with the costs of this instance against the appellant.
Villamor, Ostrand, Johns, Romualdez and Villa-Real, JJ., concur.
Republic of the Philippines
SUPREME COURT
Manila
EN BANC
G.R. No. L-12465 May 29, 1959
YU PANG CHENG alias YU PANG CHING, petitioner,
vs.
THE COURT OF APPEALS, ET AL., respondents.
M. de la Rosa and Yuseco, Abdon, Yuseco and Narvasa for petitioner.
Perkin and Ponce Enrile for respondents.
BAUTISTA ANGELO, J.:
4

Plaintiff brought this action to collect from defendant the sum of P10,000.00, value of an
insurance policy taken upon the life of one Yu Pang Eng, plus interest thereon at the legal rate,
the sum of P10,000.00 as moral damages the further sum of P3,000.00 as attorney's fees, and
the costs of action.
Defendant, in its answer, set up the defense that the insured was guilty of misrepresentation
and concealment of material facts in that he gave false and untruthful answers to certain
questions asked him in his application for insurance which were material to the risk insured
against and have the effect of avoiding the insurance policy.
After trial, the court rendered judgment ordering defendant to pay plaintiff the sum of
P10,000.00, with legal interest thereon from the filing of the complaint, plus the sum of
P2,000.00 as attorney's fees, and the costs of suit. On appeal, the Court of Appeals reversed
the decision of the trial court, holding that the insured was guilty of concealment of material
facts which relieves defendant from liability. Hence the present petition for review.
On September 5, 1950, Yu Pang Eng submitted parts II and III of his application for insurance
consisting of the medical declaration made by him to the medical examiner of defendant and
the medical examiner's report. On September 7, he submitted part I of his application which is
the declaration made by him to an agent of defendant, and on September 8, based on said
application, and upon payment of the first premium in the sum of P591.70, defendant issued
to the insured Policy No. 812858.
On December 27, 1950, the insured entered St. Luke's Hospital for medical treatment but he
died on February 27, 1951. According to the death certificate, he died of "infiltrating medullary
carcinoma, Grade 4, advanced cardiac and of lesser curvature, stomach metastases spleen."
Plaintiff, brother and beneficiary of the insured, demanded from the defendant the payment of
the proceeds of the insurance policy and when the demand was refused, he brought the
present action.
The issue to be determined is whether the insured is guilty of concealment of some facts
material to the risk insured against which has the effect of avoiding the policy as found by
respondent court.
The insured, in his application for insurance, particularly in his declarations to the examining
physician, stated the following in answering the questions propounded to him:
14. Have you ever had any of the following diseases or symptoms? Each question
must be read and answered "Yes" or "No".
x x x x x x x x x
Gastritis, Ulcer of the Stomach or any disease of that organ? No.
Vertigo, Dizziness, Fainting-spells or Unconscious? No.
Cancer, Tumors or Ulcers of any kind? No.
15. Have you ever consulted any physician not included in any of the above answers?
Give names and address or physicians list ailments or accidents and date. No.
It appears that the insured entered the Chinese General Hospital for medical treatment on
January 29, 1950 having stayed there up to February 11, 1950. Upon entering the hospital, he
complained of dizziness, anemia, abdominal pains and tarry stools, and in the evening of his
admission he had several abdominal pains and his discharges were with black tarry stools and
felt dizzy and weak. The history of his illness shows that the same "started a year ago as
frequent dizziness." An X-Ray picture of his stomach was taken and the diagnosis made of him
by his doctors showed that his illness was "peptic ulcer, bleeding."
It should be noted that the insured's confinement in the Chinese General Hospital took place
from January 29, 1950 to February 11, 1950, whereas his application for insurance wherein he
stated his answer to the questions propounded to him by the examining physician of
defendant was submitted to defendant on September 5, 1950. It is apparent that when the
insured gave his answers regarding his previous ailment, particularly with regard to "Gastritis,
Ulcer of the Stomach or any disease of that organ" and "Vertigo, Dizziness, Fainting-spells or
Unconsciousness", he concealed the ailment of which he was treated in the Chinese General,
Hospital which precisely has direct connection with the subject of the questions propounded.
The negative answers given by the insured regarding his previous ailment, or his concealment
of the fact that he was hospitalized and treated for sometime of peptic ulcer and had suffered
from "dizziness, anemia, abdominal pains and tarry stools", deprived defendant of the
opportunity to make the necessary inquiry as to the nature of his past illness so that as it may
form its estimate relative to the approval of his application. Had defendant been given such
opportunity, considering the previous illness of the insured as disclosed by the record of the
Chinese General Hospital, defendant would probably had never consented to the issuance of
the policy in question. In fact, according to the death certificate, the insured died of
"infiltrating medullary carcinoma, Grade 4, advanced cardiac and of lesser curvature, stomach
metastases spleen", which may have direct connection with his previous illness.
Our Insurance law provides that " A neglect to communicate that which a party knows and
ought to communicate, is called concealment" (Section 25, Act No. 2427). Whether intentional
or unintentional, the concealment entitles the insurer to rescind the contract of insurance
(Section 26). Our law even requires the insured to communicate to the insurer all facts within
his knowledge which are material to the contract and which the other party has not the means
of ascertaining (Section 27), and the materiality is to be determined not by the event but solely
by the probable and reasonable influence of the facts upon the party to whom the
communication is due (Section 30).
In the case of Argente vs. West Coast Life Insurance Co., 51 Phil., 725 this Court said:
One ground for the rescission of a contract of insurance under the insurance Act is "a
concealment", which in section 25 is defined "A neglect to communicate that which a
party knows and ought to communicate." Appellant argues that the concealment was
immaterial and insufficient to avoid the policy. We cannot agree. In an action on a
5

life insurance policy where the evidence conclusively shows that the answers to
questions concerning diseases were untrue, the truth or falsity of the answers
become the determining factor. If the policy was procured by fraudulent
representations, the contract of insurance apparently set forth therein was never
legally existent. It can fairly be assumed that had the true facts been disclosed by the
assured, the insurance would never have been granted.
Upon the foregoing reasons, we are persuaded to conclude that respondent court did err in
declaring the policy ineffective on the ground of concealment and in relieving appellee from
liability thereunder.
Wherefore, the decision appealed from is affirmed, with costs against petitioner-appellant.
Paras, C.J., Bengzon, Padilla, Montemayor, Reyes, A., Labrador, Concepcion and Endencia,
JJ., concur.
Republic of the Philippines
SUPREME COURT
Manila
EN BANC
G.R. No. L-16163 February 28, 1963
IGNACIO SATURNINO, in his own behalf and as the JUDICIAL GUARDIAN OF CARLOS
SATURNINO, minor,plaintiffs-appellants,
vs.
THE PHILIPPINE AMERICAN LIFE INSURANCE COMPANY, defendant-appellee.
Eleazaro A. Samson for plaintiffs-appellants.
Abello & Macias for defendant-appellee.
MAKALINTAL, J.:
Plaintiffs, now appellants, filed this action in the Court of First Instance of Manila to recover
the sum of P5,000.00, corresponding to the face value of an insurance policy issued by
defendant on the life of Estefania A. Saturnino, and the sum of P1,500.00 as attorney's fees.
Defendant, now appellee, set up special defenses in its answer, with a counterclaim for
damages allegedly sustained as a result of the unwarranted presentation of this case. Both the
complaint and the counterclaim were dismissed by the trial court; but appellants were
declared entitled to the return of the premium already paid; plus interest at 6% up to January
8, 1959, when a check for the corresponding amount P359.65 was sent to them by
appellee.
The policy sued upon is one for 20-year endowment non-medical insurance. This kind of policy
dispenses with the medical examination of the applicant usually required in ordinary life
policies. However, detailed information is called for in the application concerning the
applicant's health and medical history. The written application in this case was submitted by
Saturnino to appellee on November 16, 1957, witnessed by appellee's agent Edward A. Santos.
The policy was issued on the same day, upon payment of the first year's premium of P339.25.
On September 19, 1958 Saturnino died of pneumonia, secondary to influenza. Appellants here,
who are her surviving husband and minor child, respectively, demanded payment of the face
value of the policy. The claim was rejected and this suit was subsequently instituted.
It appears that two months prior to the issuance of the policy or on September 9, 1957,
Saturnino was operated on for cancer, involving complete removal of the right breast,
including the pectoral muscles and the glands found in the right armpit. She stayed in the
hospital for a period of eight days, after which she was discharged, although according to the
surgeon who operated on her she could not be considered definitely cured, her ailment being
of the malignant type.
Notwithstanding the fact of her operation Estefania A. Saturnino did not make a disclosure
thereof in her application for insurance. On the contrary, she stated therein that she did not
have, nor had she ever had, among other ailments listed in the application, cancer or other
tumors; that she had not consulted any physician, undergone any operation or suffered any
injury within the preceding five years; and that she had never been treated for nor did she ever
have any illness or disease peculiar to her sex, particularly of the breast, ovaries, uterus, and
menstrual disorders. The application also recites that the foregoing declarations constituted "a
further basis for the issuance of the policy."
The question at issue is whether or not the insured made such false representations of
material facts as to avoid the policy. There can be no dispute that the information given by her
in her application for insurance was false, namely, that she had never had cancer or tumors, or
consulted any physician or undergone any operation within the preceding period of five years.
Are the facts then falsely represented material? The Insurance Law (Section 30) provides that
"materiality is to be determined not by the event, but solely by the probable and reasonable
influence of the facts upon the party to whom the communication is due, in forming his
estimate of the proposed contract, or in making his inquiries." It seems to be the contention of
appellants that the facts subject of the representation were not material in view of the "non-
medical" nature of the insurance applied for, which does away with the usual requirement of
medical examination before the policy is issued. The contention is without merit. If anything,
the waiver of medical examination renders even more material the information required of the
applicant concerning previous condition of health and diseases suffered, for such information
necessarily constitutes an important factor which the insurer takes into consideration in
deciding whether to issue the policy or not. It is logical to assume that if appellee had been
properly apprised of the insured's medical history she would at least have been made to
undergo medical examination in order to determine her insurability.
Appellants argue that due information concerning the insured's previous illness and operation
had been given to appellees agent Edward A. Santos, who filled the application form after it
was signed in blank by Estefania A. Saturnino. This was denied by Santos in his testimony, and
6

the trial court found such testimony to be true. This is a finding of fact which is binding upon
us, this appeal having been taken upon questions of law alone. We do not deem it necessary,
therefore, to consider appellee's additional argument, which was upheld by the trial court, that
in signing the application form in blank and leaving it to Edward A. Santos to fill (assuming that
to be the truth) the insured in effect made Santos her agent for that purpose and consequently
was responsible for the errors in the entries made by him in that capacity.
In the application for insurance signed by the insured in this case, she agreed to submit to a
medical examination by a duly appointed examiner of appellee if in the latter's opinion such
examination was necessary as further evidence of insurability. In not asking her to submit to a
medical examination, appellants maintain, appellee was guilty of negligence, which precluded
it from finding about her actual state of health. No such negligence can be imputed to
appellee. It was precisely because the insured had given herself a clean bill of health that
appellee no longer considered an actual medical checkup necessary.
Appellants also contend there was no fraudulent concealment of the truth inasmuch as the
insured herself did not know, since her doctor never told her, that the disease for which she
had been operated on was cancer. In the first place the concealment of the fact of the
operation itself was fraudulent, as there could not have been any mistake about it, no matter
what the ailment. Secondly, in order to avoid a policy it is not necessary to show actual fraud
on the part of the insured. In the case of Kasprzyk v. Metropolitan Insurance Co., 140 N.Y.S.
211, 214, it was held:
Moreover, if it were the law that an insurance company could not depend a policy on
the ground of misrepresentation, unless it could show actual knowledge on the part
of the applicant that the statements were false, then it is plain that it would be
impossible for it to protect itself and its honest policyholders against fraudulent and
improper claims. It would be wholly at the mercy of any one who wished to apply for
insurance, as it would be impossible to show actual fraud except in the extremest
cases. It could not rely on an application as containing information on which it could
act. There would be no incentive to an applicant to tell the truth.
Wherefore, the parties respectfully pray that the foregoing stipulation of facts be
admitted and approved by this Honorable Court, without prejudice to the parties
adducing other evidence to prove their case not covered by this stipulation of
facts. 1wph1.t
In this jurisdiction a concealment, whether intentional or unintentional, entitles the insurer to
rescind the contract of insurance, concealment being defined as "negligence to communicate
that which a party knows and ought to communicate" (Sections 24 & 26, Act No. 2427). In the
case of Argente v. West Coast Life Insurance Co., 51 Phil. 725, 732, this Court said, quoting from
Joyce, The Law of Insurance, 2nd ed., Vol. 3:
"The basis of the rule vitiating the contract in cases of concealment is that it misleads
or deceives the insurer into accepting the risk, or accepting it at the rate of premium
agreed upon. The insurer, relying upon the belief that the assured will disclose every
material fact within his actual or presumed knowledge, is misled into a belief that the
circumstance withheld does not exist, and he is thereby induced to estimate the risk
upon a false basis that it does not exist."
The judgment appealed from, dismissing the complaint and awarding the return to appellants
of the premium already paid, with interest at 6% up to January 29, 1959, affirmed, with costs
against appellants.
Bengzon, C.J., Padilla, Bautista Angelo, Labrador, Concepcion, Reyes, J.B.L., Barrera, Paredes,
Dizon and Regala, JJ., concur.
SECOND DIVISION
[G.R. No. 113899. October 13, 1999]
GREAT PACIFIC LIFE ASSURANCE CORP., petitioner vs. COURT OF APPEALS AND MEDARDA V.
LEUTERIO, respondents.
D E C I S I O N
QUISUMBING, J.:
This petition for review, under Rule 45 of the Rules of Court, assails the Decision
[1]
dated
May 17, 1993, of the Court of Appeals and its Resolution
[2]
dated January 4, 1994 in CA-G.R. CV
No. 18341. The appellate court affirmed in toto the judgment of the Misamis Oriental Regional
Trial Court, Branch 18, in an insurance claim filed by private respondent against Great Pacific
Life Assurance Co. The dispositive portion of the trial courts decision reads:
WHEREFORE, judgment is rendered adjudging the defendant GREAT PACIFIC LIFE ASSURANCE
CORPORATION as insurer under its Group policy No. G-1907, in relation to Certification B-
18558 liable and ordered to pay to the DEVELOPMENT BANK OF THE PHILIPPINES as creditor of
the insured Dr. Wilfredo Leuterio, the amount of EIGHTY SIX THOUSAND TWO HUNDRED
PESOS (P86,200.00); dismissing the claims for damages, attorneys fees and litigation expenses
in the complaint and counterclaim, with costs against the defendant and dismissing the
complaint in respect to the plaintiffs, other than the widow-beneficiary, for lack of cause of
action.
[3]

The facts, as found by the Court of Appeals, are as follows:
A contract of group life insurance was executed between petitioner Great Pacific Life
Assurance Corporation (hereinafter Grepalife) and Development Bank of the Philippines
(hereinafter DBP). Grepalife agreed to insure the lives of eligible housing loan mortgagors of
DBP.
7

On November 11, 1983, Dr. Wilfredo Leuterio, a physician and a housing debtor of DBP
applied for membership in the group life insurance plan. In an application form, Dr. Leuterio
answered questions concerning his health condition as follows:
7. Have you ever had, or consulted, a physician for a heart condition, high blood pressure,
cancer, diabetes, lung, kidney or stomach disorder or any other physical impairment?
Answer: No. If so give details ___________.
8. Are you now, to the best of your knowledge, in good health?
Answer: [ x ] Yes [ ] No.
[4]

On November 15, 1983, Grepalife issued Certificate No. B-18558, as insurance coverage
of Dr. Leuterio, to the extent of his DBP mortgage indebtedness amounting to eighty-six
thousand, two hundred (P86,200.00) pesos.
On August 6, 1984, Dr. Leuterio died due to massive cerebral hemorrhage.
Consequently, DBP submitted a death claim to Grepalife. Grepalife denied the claim alleging
that Dr. Leuterio was not physically healthy when he applied for an insurance coverage on
November 15, 1983. Grepalife insisted that Dr. Leuterio did not disclose he had been suffering
from hypertension, which caused his death. Allegedly, such non-disclosure constituted
concealment that justified the denial of the claim.
On October 20, 1986, the widow of the late Dr. Leuterio, respondent Medarda V.
Leuterio, filed a complaint with the Regional Trial Court of Misamis Oriental, Branch 18, against
Grepalife for Specific Performance with Damages.
[5]
During the trial, Dr. Hernando Mejia,
who issued the death certificate, was called to testify. Dr. Mejias findings, based partly from
the information given by the respondent widow, stated that Dr. Leuterio complained of
headaches presumably due to high blood pressure. The inference was not conclusive because
Dr. Leuterio was not autopsied, hence, other causes were not ruled out.
On February 22, 1988, the trial court rendered a decision in favor of respondent widow
and against Grepalife. On May 17, 1993, the Court of Appeals sustained the trial courts
decision. Hence, the present petition. Petitioners interposed the following assigned errors:
"1. THE LOWER COURT ERRED IN HOLDING DEFENDANT-APPELLANT LIABLE TO THE
DEVELOPMENT BANK OF THE PHILIPPINES (DBP) WHICH IS NOT A PARTY TO
THE CASE FOR PAYMENT OF THE PROCEEDS OF A MORTGAGE REDEMPTION
INSURANCE ON THE LIFE OF PLAINTIFFS HUSBAND WILFREDO LEUTERIO ONE
OF ITS LOAN BORROWERS, INSTEAD OF DISMISSING THE CASE AGAINST
DEFENDANT-APPELLANT [Petitioner Grepalife] FOR LACK OF CAUSE OF ACTION.
2. THE LOWER COURT ERRED IN NOT DISMISSING THE CASE FOR WANT OF
JURISDICTION OVER THE SUBJECT OR NATURE OF THE ACTION AND OVER THE
PERSON OF THE DEFENDANT.
3. THE LOWER COURT ERRED IN ORDERING DEFENDANT-APPELLANT TO PAY TO
DBP THE AMOUNT OF P86,200.00 IN THE ABSENCE OF ANY EVIDENCE TO
SHOW HOW MUCH WAS THE ACTUAL AMOUNT PAYABLE TO DBP IN
ACCORDANCE WITH ITS GROUP INSURANCE CONTRACT WITH DEFENDANT-
APPELLANT.
4. THE LOWER COURT ERRED IN - HOLDING THAT THERE WAS NO CONCEALMENT
OF MATERIAL INFORMATION ON THE PART OF WILFREDO LEUTERIO IN HIS
APPLICATION FOR MEMBERSHIP IN THE GROUP LIFE INSURANCE PLAN
BETWEEN DEFENDANT-APPELLANT OF THE INSURANCE CLAIM ARISING FROM
THE DEATH OF WILFREDO LEUTERIO.
[6]

Synthesized below are the assigned errors for our resolution:
1. Whether the Court of Appeals erred in holding petitioner liable to DBP as
beneficiary in a group life insurance contract from a complaint filed by the
widow of the decedent/mortgagor?
2. Whether the Court of Appeals erred in not finding that Dr. Leuterio concealed
that he had hypertension, which would vitiate the insurance contract?
3. Whether the Court of Appeals erred in holding Grepalife liable in the amount of
eighty six thousand, two hundred (P86,200.00) pesos without proof of the
actual outstanding mortgage payable by the mortgagor to DBP.
Petitioner alleges that the complaint was instituted by the widow of Dr. Leuterio, not the
real party in interest, hence the trial court acquired no jurisdiction over the case. It argues that
when the Court of Appeals affirmed the trial courts judgment, Grepalife was held liable to pay
the proceeds of insurance contract in favor of DBP, the indispensable party who was not joined
in the suit.
To resolve the issue, we must consider the insurable interest in mortgaged properties
and the parties to this type of contract. The rationale of a group insurance policy of
mortgagors, otherwise known as the mortgage redemption insurance, is a device for the
protection of both the mortgagee and the mortgagor. On the part of the mortgagee, it has to
enter into such form of contract so that in the event of the unexpected demise of the
mortgagor during the subsistence of the mortgage contract, the proceeds from such insurance
will be applied to the payment of the mortgage debt, thereby relieving the heirs of the
mortgagor from paying the obligation.
[7]
In a similar vein, ample protection is given to the
mortgagor under such a concept so that in the event of death; the mortgage obligation will be
extinguished by the application of the insurance proceeds to the mortgage
indebtedness.
[8]
Consequently, where the mortgagor pays the insurance premium under the
group insurance policy, making the loss payable to the mortgagee, the insurance is on the
mortgagors interest, and the mortgagor continues to be a party to the contract. In this type of
policy insurance, the mortgagee is simply an appointee of the insurance fund, such loss-
payable clause does not make the mortgagee a party to the contract.
[9]

Section 8 of the Insurance Code provides:
Unless the policy provides, where a mortgagor of property effects insurance in his own name
providing that the loss shall be payable to the mortgagee, or assigns a policy of insurance to a
mortgagee, the insurance is deemed to be upon the interest of the mortgagor, who does not
cease to be a party to the original contract, and any act of his, prior to the loss, which would
otherwise avoid the insurance, will have the same effect, although the property is in the hands
of the mortgagee, but any act which, under the contract of insurance, is to be performed by
8

the mortgagor, may be performed by the mortgagee therein named, with the same effect as if
it had been performed by the mortgagor.
The insured private respondent did not cede to the mortgagee all his rights or interests in
the insurance, the policy stating that: In the event of the debtors death before his
indebtedness with the Creditor [DBP] shall have been fully paid, an amount to pay the
outstanding indebtedness shall first be paid to the creditor and the balance of sum assured, if
there is any, shall then be paid to the beneficiary/ies designated by the debtor.
[10]
When DBP
submitted the insurance claim against petitioner, the latter denied payment thereof,
interposing the defense of concealment committed by the insured. Thereafter, DBP collected
the debt from the mortgagor and took the necessary action of foreclosure on the residential
lot of private respondent.
[11]
In Gonzales La O vs. Yek Tong Lin Fire & Marine Ins. Co.
[12]
we
held:
Insured, being the person with whom the contract was made, is primarily the proper person
to bring suit thereon. * * * Subject to some exceptions, insured may thus sue, although the
policy is taken wholly or in part for the benefit of another person named or unnamed, and
although it is expressly made payable to another as his interest may appear or otherwise. * *
* Although a policy issued to a mortgagor is taken out for the benefit of the mortgagee and is
made payable to him, yet the mortgagor may sue thereon in his own name, especially where
the mortgagees interest is less than the full amount recoverable under the policy, * * *.
And in volume 33, page 82, of the same work, we read the following:
Insured may be regarded as the real party in interest, although he has assigned the policy for
the purpose of collection, or has assigned as collateral security any judgment he may
obtain.
[13]

And since a policy of insurance upon life or health may pass by transfer, will or
succession to any person, whether he has an insurable interest or not, and such person may
recover it whatever the insured might have recovered,
[14]
the widow of the decedent Dr.
Leuterio may file the suit against the insurer, Grepalife.
The second assigned error refers to an alleged concealment that the petitioner
interposed as its defense to annul the insurance contract. Petitioner contends that Dr.
Leuterio failed to disclose that he had hypertension, which might have caused his
death. Concealment exists where the assured had knowledge of a fact material to the risk, and
honesty, good faith, and fair dealing requires that he should communicate it to the assured,
but he designedly and intentionally withholds the same.
[15]

Petitioner merely relied on the testimony of the attending physician, Dr. Hernando
Mejia, as supported by the information given by the widow of the decedent. Grepalife asserts
that Dr. Mejias technical diagnosis of the cause of death of Dr. Leuterio was a duly
documented hospital record, and that the widows declaration that her husband had possible
hypertension several years ago should not be considered as hearsay, but as part of res gestae.
On the contrary the medical findings were not conclusive because Dr. Mejia did not
conduct an autopsy on the body of the decedent. As the attending physician, Dr. Mejia stated
that he had no knowledge of Dr. Leuterios any previous hospital confinement.
[16]
Dr. Leuterios
death certificate stated that hypertension was only the possible cause of death. The private
respondents statement, as to the medical history of her husband, was due to her unreliable
recollection of events. Hence, the statement of the physician was properly considered by the
trial court as hearsay.
The question of whether there was concealment was aptly answered by the appellate
court, thus:
The insured, Dr. Leuterio, had answered in his insurance application that he was in good
health and that he had not consulted a doctor or any of the enumerated ailments, including
hypertension; when he died the attending physician had certified in the death certificate that
the former died of cerebral hemorrhage, probably secondary to hypertension. From this
report, the appellant insurance company refused to pay the insurance claim. Appellant alleged
that the insured had concealed the fact that he had hypertension.
Contrary to appellants allegations, there was no sufficient proof that the insured had suffered
from hypertension. Aside from the statement of the insureds widow who was not even sure if
the medicines taken by Dr. Leuterio were for hypertension, the appellant had not proven nor
produced any witness who could attest to Dr. Leuterios medical history...
x x x
Appellant insurance company had failed to establish that there was concealment made by the
insured, hence, it cannot refuse payment of the claim.
[17]

The fraudulent intent on the part of the insured must be established to entitle the
insurer to rescind the contract.
[18]
Misrepresentation as a defense of the insurer to avoid
liability is an affirmative defense and the duty to establish such defense by satisfactory and
convincing evidence rests upon the insurer.
[19]
In the case at bar, the petitioner failed to clearly
and satisfactorily establish its defense, and is therefore liable to pay the proceeds of the
insurance.
And that brings us to the last point in the review of the case at bar. Petitioner claims that
there was no evidence as to the amount of Dr. Leuterios outstanding indebtedness to DBP at
the time of the mortgagors death. Hence, for private respondents failure to establish the
same, the action for specific performance should be dismissed. Petitioners claim is without
merit. A life insurance policy is a valued policy.
[20]
Unless the interest of a person insured is
susceptible of exact pecuniary measurement, the measure of indemnity under a policy of
insurance upon life or health is the sum fixed in the policy.
[21]
The mortgagor paid the premium
according to the coverage of his insurance, which states that:
The policy states that upon receipt of due proof of the Debtors death during the terms of this
insurance, a death benefit in the amount of P86,200.00 shall be paid.
In the event of the debtors death before his indebtedness with the creditor shall have been
fully paid, an amount to pay the outstanding indebtedness shall first be paid to the Creditor
9

and the balance of the Sum Assured, if there is any shall then be paid to the beneficiary/ies
designated by the debtor.
[22]
(Emphasis omitted)
However, we noted that the Court of Appeals decision was promulgated on May 17,
1993. In private respondents memorandum, she states that DBP foreclosed in 1995 their
residential lot, in satisfaction of mortgagors outstanding loan. Considering this supervening
event, the insurance proceeds shall inure to the benefit of the heirs of the deceased person or
his beneficiaries. Equity dictates that DBP should not unjustly enrich itself at the expense of
another (Nemo cum alterius detrimenio protest). Hence, it cannot collect the insurance
proceeds, after it already foreclosed on the mortgage. The proceeds now rightly belong to Dr.
Leuterios heirs represented by his widow, herein private respondent Medarda Leuterio.
WHEREFORE, the petition is hereby DENIED. The Decision and Resolution of the Court of
Appeals in CA-G.R. CV 18341 is AFFIRMED with MODIFICATION that the petitioner is ORDERED
to pay the insurance proceeds amounting to Eighty-six thousand, two hundred (P86,200.00)
pesos to the heirs of the insured, Dr. Wilfredo Leuterio (deceased), upon presentation of proof
of prior settlement of mortgagors indebtedness to Development Bank of the
Philippines. Costs against petitioner.
SO ORDERED.
Mendoza, Buena, and De Leon Jr., JJ., concur.
Bellosillo, (Chairman), J., on official leave.
Republic of the Philippines
SUPREME COURT
Manila
EN BANC
G.R. No. L-14373 January 30, 1960
GENERAL INSURANCE AND SURETY CORPORATION, petitioner,
vs.
NG HUA, respondent.
Jose P. Bengzon, Guido Advincula and Potenciano Villegas, Jr., petitioner.
Crispin D. Baizas for respondent.
BENGZON, J.:
Suit to recover on a fire insurance policy. The insurer presented several defenses in the Manila
court of first instance. After trial, it was required to pay.
On appeal to the Courts of Appeal, the judgment was affirmed.
This is now a revision on certiorari, upon the insurer's insistence on two of its main defenses:
prescription and breach of warranty.
The principal of facts on which adjudication may rest are these:
On April 15, 1952, the defendant General Insurance and Surety Corporation issued its
insurance Policy No. 471, insuring against fire, for one year, the stock in trade of the Central
Pomade Factory owned by Ng Hua, the court insured. The next day, the Pomade factory
building burned, resulting in destruction by fire of the insured properties. Ng Hua claimed
indemnity from the insurer. The policy covered damages up to P10,000.00; but after some
negotiations and upon suggestion of the Manila Adjustment Company, he reduced the claim of
P5,000.00. Nevertheless, the defendant insurer refused to pay for various reasons, namely (a)
action was not filed in time; (b) violation of warranty; (c) submission of fraudulent claim; and
(f) failure to pay the premium.
The aforesaid Policy No. 471 contains this stipulation on the back thereof;.
3. The insured shall give notice to the company of any insurance or insurances
already affected, or which may subsequently be effected, covering any of the
property hereby insured, and unless such notice be given and the particulars of such
insurance or insurances be stated in or endorsed on this Policy by or on behalf of the
Company before the occurrence of any loss or damage, all benefits under the policy
shall be forfeited. (Emphasis ours.)
The face of the policy bore the annotation: "Co-Insurance Declared NIL"
It is undenied that Ng Hua had obtained fire insurance on the same goods, for the same period
of time, in the amount of P20,000.00 from General Indemnity Co. However, the Court of
Appeals referring to the annotation and overruling the defense, held that there was no
violation of the above clause, inasmuch as "co-insurance exists when a condition of the policy
requires the insured to bear ratable proportion of the loss when the value of the insured
property exceeds the face value of the policy," hence there is no co-insurance here.
Discussion Undoubtedly, co-insurance exists under the condition described by the appellate
court. But that isone kind of co-insurance. It is not the only situation where co-insurance exists.
Other insurers of the same property against the same hazard are sometimes referred as co-
insurers and the ensuing combination as co-insurance.
1
And considering the terms of the policy
which required the insured to declare other insurances, the statement in question must be
deemed to be a statement (warranty) binding on both insurer and insured, that there were no
other insurance on the property. Remember it runs "Co-Insurance declared"; emphasis on the
last word. If "Co-Insurance" means that the Court of Appeals says, the annotation served no
purpose. It would even becontrary to the policy itself, which in its clause No. 17 made the
insured a co-insurer for the excess of the value of the property over the amount of the policy.
The annotation then, must be deemed to be a warranty that the property was not insured by
any other policy. Violation thereof entitles the insurer to rescind. (Sec. 69. Insurance Act) Such
10

misrepresentation is fatal in the light of our views in Santa Ana vs. Commercial Union
Assurance Company, Ltd., 55 Phil., 329. The materiality of non-disclosure of other insurance
policies is not open to doubt.
Furthermore, even if the annotations were overlooked, the defendant insurer would still be
free from liability because there is no question that the policy issued by General Indemnity had
not been stated in nor endorsed onPolicy No. 471 of defendant. And as stipulated in the above-
quoted provisions of such policy "all benefit under this policy shall be forfeited."
2

To avoid the dissastrous effect of the misrepresentation or concealment of the other insurance
policy, Ng Hua alleges "actual knowledge" on the part of General insurance of the fact that he
had taken out additional insurance with General Indemnity. He does not say when such
knowledge was acquired or imparted. If General Insurance know before issuing its policy
or before the fire, such knowledge might overcome the insurer's defense.
3
However, the Court
of Appeals found no evidence of such knowledge. We have read the pages of the stenographic
notes cited by Ng Hua and we all gather is evidence of the existence of the Insurance General
Indemnity Company. As to knowledge of General Insurance before issuance of its policy or the
fire, there was none.
Indeed, this concealment and violation was expressly set up as a special defense in the answer.
Yet plaintiff did not, in avoidance, reply nor assert such knowledge. And it is doubtful whether
the evidence on the point would be admissible under the pleadings. (See Rule 11, sec. 1.)
All the above considerations lead to the conclusion that the defendant insurer successfully
established its defense of warranty breach or concealment of the other insurance and/or
violation of the provision of the policy above-mentioned.
Having reached the conclusion, we deem it unnecessary to discuss the other defenses.
Wherefore, the judgment under review will be revoked, and the defendant insurer (herein
petitioner) acquitted from all the liability under the policy. Costs against respondent. So
ordered.
Paras, C.J., Padilla, Montemayor, Bautista Angelo, Labrador, Concepcion Reyes, J.B.L., Endencia,
and Barrera, JJ., concur.
Republic of the Philippines
SUPREME COURT
Manila
THIRD DIVISION

G.R. No. 92492 June 17, 1993
THELMA VDA. DE CANILANG, petitioner,
vs.
HON. COURT OF APPEALS and GREAT PACIFIC LIFE ASSURANCE CORPORATION, respondents.
Simeon C. Sato for petitioner.
FELICIANO, J.:
On 18 June 1982, Jaime Canilang consulted Dr. Wilfredo B. Claudio and was diagnosed as
suffering from "sinus tachycardia." The doctor prescribed the following fro him: Trazepam, a
tranquilizer; and Aptin, a beta-blocker drug. Mr. Canilang consulted the same doctor again on 3
August 1982 and this time was found to have "acute bronchitis."
On next day, 4 August 1982, Jaime Canilang applied for a "non-medical" insurance policy with
respondent Great Pacific Life Assurance Company ("Great Pacific") naming his wife, Thelma
Canilang, as his beneficiary.
1
Jaime Canilang was issued ordinary life insurance Policy No.
345163, with the face value of P19,700, effective as of 9 August 1982.
On 5 August 1983, Jaime Canilang died of "congestive heart failure," "anemia," and "chronic
anemia."
2
Petitioner, widow and beneficiary of the insured, filed a claim with Great Pacific
which the insurer denied on 5 December 1983 upon the ground that the insured had concealed
material information from it.
Petitioner then filed a complaint against Great Pacific with the Insurance Commission for
recovery of the insurance proceeds. During the hearing called by the Insurance Commissioner,
petitioner testified that she was not aware of any serious illness suffered by her late
husband
3
and that, as far as she knew, her husband had died because of a kidney disorder.
4
A
deposition given by Dr. Wilfredo Claudio was presented by petitioner. There Dr. Claudio stated
that he was the family physician of the deceased Jaime Canilang
5
and that he had previously
treated him for "sinus tachycardia" and "acute bronchitis."
6
Great Pacific for its part presented
Dr. Esperanza Quismorio, a physician
and a medical underwriter working for Great Pacific.
7
She testified that the deceased's
insurance application had been approved on the basis of his medical declaration.
8
She
explained that as a rule, medical examinations are required only in cases where the applicant
has indicated in his application for insurance coverage that he has previously undergone
medical consultation and hospitalization.
9

In a decision dated 5 November 1985, Insurance Commissioner Armando Ansaldo ordered
Great Pacific to pay P19,700 plus legal interest and P2,000.00 as attorney's fees after holding
that:
1. the ailment of Jaime Canilang was not so serious that, even if it had been
disclosed, it would not have affected Great Pacific's decision to insure him;
11

2. Great Pacific had waived its right to inquire into the health condition of
the applicant by the issuance of the policy despite the lack of answers to
"some of the pertinent questions" in the insurance application;
3. there was no intentional concealment on the part of the insured Jaime
Canilang as he had thought that he was merely suffering from a minor
ailment and simple cold;
10
and
4. Batas Pambansa Blg. 847 which voids an insurance contract, whether or
not concealment was intentionally made, was not applicable to Canilang's
case as that law became effective only on 1 June 1985.
On appeal by Great Pacific, the Court of Appeals reversed and set aside the decision of the
Insurance Commissioner and dismissed Thelma Canilang's complaint and Great Pacific's
counterclaim. The Court of Appealed found that the use of the word "intentionally" by the
Insurance Commissioner in defining and resolving the issue agreed upon by the parties at pre-
trial before the Insurance Commissioner was not supported by the evidence; that the issue
agreed upon by the parties had been whether the deceased insured, Jaime Canilang, made
a material concealment as the state of his health at the time of the filing of insurance
application, justifying respondent's denial of the claim. The Court of Appeals also found that the
failure of Jaime Canilang to disclose previous medical consultation and treatment constituted
material information which should have been communicated to Great Pacific to enable the
latter to make proper inquiries. The Court of Appeals finally held that the Ng Gan Zee case
which had involved misrepresentation was not applicable in respect of the case at bar which
involves concealment.
Petitioner Thelma Canilang is now before this Court on a Petition for Review
on Certiorari alleging that:
1. . . . the Honorable Court of Appeals, speaking with due respect, erred in
not holding that the issue in the case agreed upon between the parties
before the Insurance Commission is whether or not Jaime Canilang
"intentionally" made material concealment in stating his state of health;
2. . . . at any rate, the non-disclosure of certain facts about his previous
health conditions does not amount to fraud and private respondent is
deemed to have waived inquiry thereto.
11

The medical declaration which was set out in the application for insurance executed by Jaime
Canilang read as follows:
MEDICAL DECLARATION
I hereby declare that:
(1) I have not been confined in any hospital, sanitarium or infirmary,
nor receive any medical or surgical advice/attention within the last five (5)
years.
(2) I have never been treated nor consulted a physician for a heart
condition, high blood pressure, cancer, diabetes, lung, kidney, stomach
disorder, or any other physical impairment.
(3) I am, to the best of my knowledge, in good health.
EXCEPTIONS:
_____________________________________________________________
___________________
GENERAL DECLARATION
I hereby declare that all the foregoing answers and statements are
complete, true and correct. I herebyagree that if there be any fraud or
misrepresentation in the above statements material to the risk, the
INSURANCE COMPANY upon discovery within two (2) years from the
effective date of insurance shall have the right to declare such insurance
null and void. That the liabilities of the Company under the said
Policy/TA/Certificate shall accrue and begin only from the date of
commencement of risk stated in the Policy/TA/Certificate, provided that the
first premium is paid and the Policy/TA/Certificate is delivered to, and
accepted by me in person, when I am in actual good health.
Signed at Manila his 4th day of August, 1992.
I
l
l
e
g
i
b
l
e

12


S
i
g
n
a
t
u
r
e

o
f

A
p
p
l
i
c
a
n
t
.

1
2

We note that in addition to the negative statements made by Mr. Canilang in paragraph 1 and
2 of the medical declaration, he failed to disclose in the appropriate space, under the caption
"Exceptions," that he had twice consulted Dr. Wilfredo B. Claudio who had found him to be
suffering from "sinus tachycardia" and "acute bronchitis."
The relevant statutory provisions as they stood at the time Great Pacific issued the contract of
insurance and at the time Jaime Canilang died, are set out in P.D. No. 1460, also known as the
Insurance Code of 1978, which went into effect on 11 June 1978. These provisions read as
follows:
Sec. 26. A neglect to communicate that which a party knows and ought to
communicate, is called a concealment.
xxx xxx xxx
Sec. 28. Each party to a contract of insurance must communicate to the
other, in good faith, all factorswithin his knowledge which are material to
the contract and as to which he makes no warranty, and which the other
has not the means of ascertaining. (Emphasis supplied)
Under the foregoing provisions, the information concealed must be information which the
concealing party knew and "ought to [have] communicate[d]," that is to say, information which
was "material to the contract." The test of materiality is contained in Section 31 of the
Insurance Code of 1978 which reads:
Sec. 31. Materially is to be determined not by the event, but solely by
the probable and reasonable influence of the facts upon the party to whom
the communication is due, in forming his estimate of the disadvantages of
the proposed contract, or in making his inquiries. (Emphasis supplied)
"Sinus tachycardia" is considered present "when the heart rate exceeds 100 beats per
minute."
13
The symptoms of this condition include pounding in the chest and sometimes
faintness and weakness of the person affected. The following elaboration was offered by Great
Pacific and set out by the Court of Appeals in its Decision:
Sinus tachycardia is defined as sinus-initiated; heart rate faster than 100
beats per minute. (Harrison' s Principles of Internal Medicine, 8th ed.
[1978], p. 1193.) It is, among others, a common reaction to heart disease,
including myocardial infarction, and heart failure per se. (Henry J.L. Marriot,
M.D.,Electrocardiography, 6th ed., [1977], p. 127.) The medication
prescribed by Dr. Claudio for treatment of Canilang's ailment on June 18,
1982, indicates the condition that said physician was trying to manage.
Thus, he prescribed Trazepam, (Philippine Index of Medical
Specialties (PIMS), Vol. 14, No. 3, Dec. 1985, p. 112) which is anti-anxiety,
anti-convulsant, muscle-relaxant; and Aptin, (Idem, p. 36) a cardiac drug,
for palpitations and nervous heart. Such treatment could have been a very
material information to the insurer in determining the action to be take on
Canilang's application for life insurance coverage.
14

We agree with the Court of Appeals that the information which Jaime Canilang failed to
disclose was material to the ability of Great Pacific to estimate the probable risk he presented
as a subject of life insurance. Had Canilang disclosed his visits to his doctor, the diagnosis made
and medicines prescribed by such doctor, in the insurance application, it may be reasonably
assumed that Great Pacific would have made further inquiries and would have probably refused
to issue a non-medical insurance policy or, at the very least, required a higher premium for the
same coverage.
15
The materiality of the information withheld by Great Pacific did not depend
upon the state of mind of Jaime Canilang. A man's state of mind or subjective belief is not
capable of proof in our judicial process, except through proof of external acts or failure to act
from which inferences as to his subjective belief may be reasonably drawn. Neither does
materiality depend upon the actual or physical events which ensue. Materiality relates rather to
the "probable and reasonable influence of the facts" upon the party to whom the
communication should have been made, in assessing the risk involved in making or omitting to
13

make further inquiries and in accepting the application for insurance; that "probable and
reasonable influence of the facts" concealed must, of course, be determined objectively, by the
judge ultimately.
The insurance Great Pacific applied for was a "non-medical" insurance policy. In Saturnino v.
Philippine-American Life Insurance Company,
16
this Court held that:
. . . if anything, the waiver of medical examination [in a non-medical
insurance contract] renders even more material the information required of
the applicant concerning previous condition of health and diseases suffered,
for such information necessarily constitutes an important factor which the
insurer takes into consideration in deciding whether to issue the policy or
not . . . .
17
(Emphasis supplied)
The Insurance Commissioner had also ruled that the failure of Great Pacific to convey certain
information to the insurer was not "intentional" in nature, for the reason that Jaime Canilang
believed that he was suffering from minor ailment like a common cold. Section 27 of the
Insurance Code of 1978 as it existed from 1974 up to 1985, that is, throughout the time range
material for present purposes, provided that:
Sec. 27. A concealment entitles the injured party to rescind a contract of
insurance.
The preceding statute, Act No. 2427, as it stood from 1914 up to 1974, had provided:
Sec. 26. A concealment, whether intentional or unintentional, entitles the
injured party to rescind a contract of insurance. (Emphasis supplied)
Upon the other hand, in 1985, the Insurance Code of 1978 was amended by
B.P. Blg. 874. This subsequent statute modified Section 27 of the Insurance Code of 1978 so as
to read as follows:
Sec. 27. A concealment whether intentional or unintentional entitles the
injured party to rescind a contract of insurance. (Emphasis supplied)
The unspoken theory of the Insurance Commissioner appears to have been that by deleting the
phrase "intentional or unintentional," the Insurance Code of 1978 (prior to its amendment by
B.P. Blg. 874) intended to limit the kinds of concealment which generate a right to rescind on
the part of the injured party to "intentional concealments." This argument is not persuasive. As
a simple matter of grammar, it may be noted that "intentional" and "unintentional" cancel
each other out. The net result therefore of the phrase "whether intentional or unitentional" is
precisely to leave unqualified the term "concealment." Thus, Section 27 of the Insurance Code
of 1978 is properly read as referring to "any concealment" without regard to whether such
concealment is intentional or unintentional. The phrase "whether intentional or unintentional"
was in fact superfluous. The deletion of the phrase "whether intentional or unintentional"
could not have had the effect of imposing an affirmative requirement that a concealment must
be intentional if it is to entitle the injured party to rescind a contract of insurance. The
restoration in 1985 by B.P. Blg. 874 of the phrase "whether intentional or unintentional" merely
underscored the fact that all throughout (from 1914 to 1985), the statute did not require proof
that concealment must be "intentional" in order to authorize rescission by the injured party.
In any case, in the case at bar, the nature of the facts not conveyed to the insurer was such that
the failure to communicate must have been intentional rather than merely inadvertent. For
Jaime Canilang could not have been unaware that his heart beat would at times rise to high
and alarming levels and that he had consulted a doctor twice in the two (2) months before
applying for non-medical insurance. Indeed, the last medical consultation took place just the
day before the insurance application was filed. In all probability, Jaime Canilang went to visit
his doctor precisely because of the discomfort and concern brought about by his experiencing
"sinus tachycardia."
We find it difficult to take seriously the argument that Great Pacific had waived inquiry into the
concealment by issuing the insurance policy notwithstanding Canilang's failure to set out
answers to some of the questions in the insurance application. Such failure precisely constituted
concealment on the part of Canilang. Petitioner's argument, if accepted, would obviously erase
Section 27 from the Insurance Code of 1978.
It remains only to note that the Court of Appeals finding that the parties had not agreed in the
pretrial before the Insurance Commission that the relevant issue was whether or not Jaime
Canilang had intentionally concealed material information from the insurer, was supported by
the evidence of record, i.e., the Pre-trial Order itself dated 17 October 1984 and the Minutes of
the Pre-trial Conference dated 15 October 1984, which "readily shows that the word
"intentional" does not appear in the statement or definition of the issue in the said Order and
Minutes."
18

WHEREFORE, the Petition for Review is DENIED for lack of merit and the Decision of the Court of
Appeals dated 16 October 1989 in C.A.-G.R. SP No. 08696 is hereby AFFIRMED. No
pronouncement as to the costs.
SO ORDERED.
Bidin, Davide, Jr., Romero and Melo, JJ., concur.

Republic of the Philippines
SUPREME COURT
Manila
FIRST DIVISION

14

G.R. No. 105135 June 22, 1995
SUNLIFE ASSURANCE COMPANY OF CANADA, petitioner,
vs.
The Hon. COURT OF APPEALS and Spouses ROLANDO and BERNARDA BACANI, respondents.

QUIASON, J.:
This is a petition for review for certiorari under Rule 45 of the Revised Rules of Court to reverse
and set aside the Decision dated February 21, 1992 of the Court of Appeals in CA-G.R. CV No.
29068, and its Resolution dated April 22, 1992, denying reconsideration thereof.
We grant the petition.
I
On April 15, 1986, Robert John B. Bacani procured a life insurance contract for himself from
petitioner. He was issued Policy No. 3-903-766-X valued at P100,000.00, with double indemnity
in case of accidental death. The designated beneficiary was his mother, respondent Bernarda
Bacani.
On June 26, 1987, the insured died in a plane crash. Respondent Bernarda Bacani filed a claim
with petitioner, seeking the benefits of the insurance policy taken by her son. Petitioner
conducted an investigation and its findings prompted it to reject the claim.
In its letter, petitioner informed respondent Bernarda Bacani, that the insured did not disclose
material facts relevant to the issuance of the policy, thus rendering the contract of insurance
voidable. A check representing the total premiums paid in the amount of P10,172.00 was
attached to said letter.
Petitioner claimed that the insured gave false statements in his application when he answered
the following questions:
5. Within the past 5 years have you:
a) consulted any doctor or other health practitioner?
b) submitted to:
EGG?
X-rays?
blood tests?
other tests?
c) attended or been admitted to any hospital or other
medical facility?
6. Have you ever had or sought advice for:
xxx xxx xxx
b) urine, kidney or bladder disorder? (Rollo, p. 53)
The deceased answered question No. 5(a) in the affirmative but limited his answer to a
consultation with a certain Dr. Reinaldo D. Raymundo of the Chinese General Hospital on
February 1986, for cough and flu complications. The other questions were answered in the
negative (Rollo, p. 53).
Petitioner discovered that two weeks prior to his application for insurance, the insured was
examined and confined at the Lung Center of the Philippines, where he was diagnosed for
renal failure. During his confinement, the deceased was subjected to urinalysis, ultra-
sonography and hematology tests.
On November 17, 1988, respondent Bernarda Bacani and her husband, respondent Rolando
Bacani, filed an action for specific performance against petitioner with the Regional Trial Court,
Branch 191, Valenzuela, Metro Manila. Petitioner filed its answer with counterclaim and a list
of exhibits consisting of medical records furnished by the Lung Center of the Philippines.
On January 14, 1990, private respondents filed a "Proposed Stipulation with Prayer for
Summary Judgment" where they manifested that they "have no evidence to refute the
documentary evidence of concealment/misrepresentation by the decedent of his health
condition (Rollo, p. 62).
Petitioner filed its Request for Admissions relative to the authenticity and due execution of
several documents as well as allegations regarding the health of the insured. Private
respondents failed to oppose said request or reply thereto, thereby rendering an admission of
the matters alleged.
Petitioner then moved for a summary judgment and the trial court decided in favor of private
respondents. The dispositive portion of the decision is reproduced as follows:
WHEREFORE, judgment is hereby rendered in favor of the plaintiffs and
against the defendant, condemning the latter to pay the former the
amount of One Hundred Thousand Pesos (P100,000.00) the face value of
insured's Insurance Policy No. 3903766, and the Accidental Death Benefit
in the amount of One Hundred Thousand Pesos (P100,000.00) and further
15

sum of P5,000.00 in the concept of reasonable attorney's fees and costs of
suit.
Defendant's counterclaim is hereby Dismissed (Rollo, pp. 43-44).
In ruling for private respondents, the trial court concluded that the facts concealed by the
insured were made in good faith and under a belief that they need not be disclosed. Moreover,
it held that the health history of the insured was immaterial since the insurance policy was
"non-medical".
Petitioner appealed to the Court of Appeals, which affirmed the decision of the trial court. The
appellate court ruled that petitioner cannot avoid its obligation by claiming concealment
because the cause of death was unrelated to the facts concealed by the insured. It also
sustained the finding of the trial court that matters relating to the health history of the insured
were irrelevant since petitioner waived the medical examination prior to the approval and
issuance of the insurance policy. Moreover, the appellate court agreed with the trial court that
the policy was "non-medical" (Rollo, pp. 4-5).
Petitioner's motion for reconsideration was denied; hence, this petition.
II
We reverse the decision of the Court of Appeals.
The rule that factual findings of the lower court and the appellate court are binding on this
Court is not absolute and admits of exceptions, such as when the judgment is based on a
misappreciation of the facts (Geronimo v. Court of Appeals, 224 SCRA 494 [1993]).
In weighing the evidence presented, the trial court concluded that indeed there was
concealment and misrepresentation, however, the same was made in "good faith" and the
facts concealed or misrepresented were irrelevant since the policy was "non-medical". We
disagree.
Section 26 of The Insurance Code is explicit in requiring a party to a contract of insurance to
communicate to the other, in good faith, all facts within his knowledge which are material to
the contract and as to which he makes no warranty, and which the other has no means of
ascertaining. Said Section provides:
A neglect to communicate that which a party knows and ought to
communicate, is called concealment.
Materiality is to be determined not by the event, but solely by the probable and reasonable
influence of the facts upon the party to whom communication is due, in forming his estimate
of the disadvantages of the proposed contract or in making his inquiries (The Insurance Code,
Sec. 31).
The terms of the contract are clear. The insured is specifically required to disclose to the
insurer matters relating to his health.
The information which the insured failed to disclose were material and relevant to the
approval and issuance of the insurance policy. The matters concealed would have definitely
affected petitioner's action on his application, either by approving it with the corresponding
adjustment for a higher premium or rejecting the same. Moreover, a disclosure may have
warranted a medical examination of the insured by petitioner in order for it to reasonably
assess the risk involved in accepting the application.
In Vda. de Canilang v. Court of Appeals, 223 SCRA 443 (1993), we held that materiality of the
information withheld does not depend on the state of mind of the insured. Neither does it
depend on the actual or physical events which ensue.
Thus, "goad faith" is no defense in concealment. The insured's failure to disclose the fact that
he was hospitalized for two weeks prior to filing his application for insurance, raises grave
doubts about his bonafides. It appears that such concealment was deliberate on his part.
The argument, that petitioner's waiver of the medical examination of the insured debunks the
materiality of the facts concealed, is untenable. We reiterate our ruling in Saturnino v.
Philippine American Life Insurance Company, 7 SCRA 316 (1963), that " . . . the waiver of a
medical examination [in a non-medical insurance contract] renders even more material the
information required of the applicant concerning previous condition of health and diseases
suffered, for such information necessarily constitutes an important factor which the insurer
takes into consideration in deciding whether to issue the policy or not . . . "
Moreover, such argument of private respondents would make Section 27 of the Insurance
Code, which allows the injured party to rescind a contract of insurance where there is
concealment, ineffective (See Vda. de Canilang v. Court of Appeals, supra).
Anent the finding that the facts concealed had no bearing to the cause of death of the insured,
it is well settled that the insured need not die of the disease he had failed to disclose to the
insurer. It is sufficient that his non-disclosure misled the insurer in forming his estimates of the
risks of the proposed insurance policy or in making inquiries (Henson v. The Philippine
American Life Insurance Co., 56 O.G. No. 48 [1960]).
We, therefore, rule that petitioner properly exercised its right to rescind the contract of
insurance by reason of the concealment employed by the insured. It must be emphasized that
rescission was exercised within the two-year contestability period as recognized in Section 48
of The Insurance Code.
WHEREFORE, the petition is GRANTED and the Decision of the Court of Appeals is REVERSED
and SET ASIDE.
SO ORDERED.
16

Padilla, Davide, Jr., Bellosillo and Kapunan, JJ., concur.
Republic of the Philippines
SUPREME COURT
Manila
EN BANC
G.R. No. L-24833 September 23, 1968
FIELDMEN'S INSURANCE CO., INC., petitioner,
vs.
MERCEDES VARGAS VDA. DE SONGCO, ET AL. and COURT OF APPEALS, respondents.
Jose S. Suarez for petitioner.
Eligio G. Lagman for respondents.

FERNANDO, J.:
An insurance firm, petitioner Fieldmen's Insurance Co., Inc., was not allowed to escape liability
under a common carrier insurance policy on the pretext that what was insured, not once but
twice, was a private vehicle and not a common carrier, the policy being issued upon the
insistence of its agent who discounted fears of the insured that his privately owned vehicle
might not fall within its terms, the insured moreover being "a man of scant education,"
finishing only the first grade. So it was held in a decision of the lower court thereafter affirmed
by respondent Court of Appeals. Petitioner in seeking the review of the above decision of
respondent Court of Appeals cannot be so sanguine as to entertain the belief that a different
outcome could be expected. To be more explicit, we sustain the Court of Appeals.
The facts as found by respondent Court of Appeals, binding upon us, follow: "This is a peculiar
case. Federico Songco of Floridablanca, Pampanga, a man of scant education being only a first
grader ..., owned a private jeepney with Plate No. 41-289 for the year 1960. On September 15,
1960, as such private vehicle owner, he was induced by Fieldmen's Insurance Company
Pampanga agent Benjamin Sambat to apply for a Common Carrier's Liability Insurance Policy
covering his motor vehicle ... Upon paying an annual premium of P16.50, defendant Fieldmen's
Insurance Company, Inc. issued on September 19, 1960, Common Carriers Accident Insurance
Policy No. 45-HO- 4254 ... the duration of which will be for one (1) year, effective September
15, 1960 to September 15, 1961. On September 22, 1961, the defendant company, upon
payment of the corresponding premium, renewed the policy by extending the coverage from
October 15, 1961 to October 15, 1962. This time Federico Songco's private jeepney carried
Plate No. J-68136-Pampanga-1961. ... On October 29, 1961, during the effectivity of the
renewed policy, the insured vehicle while being driven by Rodolfo Songco, a duly licensed
driver and son of Federico (the vehicle owner) collided with a car in the municipality of
Calumpit, province of Bulacan, as a result of which mishap Federico Songco (father) and
Rodolfo Songco (son) died, Carlos Songco (another son), the latter's wife, Angelita Songco, and
a family friend by the name of Jose Manuel sustained physical injuries of varying degree."
1

It was further shown according to the decision of respondent Court of Appeals: "Amor Songco,
42-year-old son of deceased Federico Songco, testifying as witness, declared that when
insurance agent Benjamin Sambat was inducing his father to insure his vehicle, he butted in
saying: 'That cannot be, Mr. Sambat, because our vehicle is an "owner" private vehicle and not
for passengers,' to which agent Sambat replied: 'whether our vehicle was an "owner" type or
for passengers it could be insured because their company is not owned by the Government
and the Government has nothing to do with their company. So they could do what they please
whenever they believe a vehicle is insurable' ... In spite of the fact that the present case was
filed and tried in the CFI of Pampanga, the defendant company did not even care to rebut
Amor Songco's testimony by calling on the witness-stand agent Benjamin Sambat, its
Pampanga Field Representative."
2

The plaintiffs in the lower court, likewise respondents here, were the surviving widow and
children of the deceased Federico Songco as well as the injured passenger Jose Manuel. On the
above facts they prevailed, as had been mentioned, in the lower court and in the respondent
Court of Appeals.1awphl.nt
The basis for the favorable judgment is the doctrine announced in Qua Chee Gan v. Law Union
and Rock Insurance Co., Ltd.,
3
with Justice J. B. L. Reyes speaking for the Court. It is now
beyond question that where inequitable conduct is shown by an insurance firm, it is "estopped
from enforcing forfeitures in its favor, in order to forestall fraud or imposition on the
insured."
4

As much, if not much more so than the Qua Chee Gan decision, this is a case where the
doctrine of estoppel undeniably calls for application. After petitioner Fieldmen's Insurance Co.,
Inc. had led the insured Federico Songco to believe that he could qualify under the common
carrier liability insurance policy, and to enter into contract of insurance paying the premiums
due, it could not, thereafter, in any litigation arising out of such representation, be permitted
to change its stand to the detriment of the heirs of the insured. As estoppel is primarily based
on the doctrine of good faith and the avoidance of harm that will befall the innocent party due
to its injurious reliance, the failure to apply it in this case would result in a gross travesty of
justice.
That is all that needs be said insofar as the first alleged error of respondent Court of Appeals is
concerned, petitioner being adamant in its far-from-reasonable plea that estoppel could not be
invoked by the heirs of the insured as a bar to the alleged breach of warranty and condition in
the policy. lt would now rely on the fact that the insured owned a private vehicle, not a
common carrier, something which it knew all along when not once but twice its agent, no
doubt without any objection in its part, exerted the utmost pressure on the insured, a man of
scant education, to enter into such a contract.
Nor is there any merit to the second alleged error of respondent Court that no legal liability
was incurred under the policy by petitioner. Why liability under the terms of the policy
5
was
inescapable was set forth in the decision of respondent Court of Appeals. Thus: "Since some of
17

the conditions contained in the policy issued by the defendant-appellant were impossible to
comply with under the existing conditions at the time and 'inconsistent with the known facts,'
the insurer 'is estopped from asserting breach of such conditions.' From this jurisprudence, we
find no valid reason to deviate and consequently hold that the decision appealed from should
be affirmed. The injured parties, to wit, Carlos Songco, Angelito Songco and Jose Manuel, for
whose hospital and medical expenses the defendant company was being made liable, were
passengers of the jeepney at the time of the occurrence, and Rodolfo Songco, for whose burial
expenses the defendant company was also being made liable was the driver of the vehicle in
question. Except for the fact, that they were not fare paying passengers, their status as
beneficiaries under the policy is recognized therein."
6

Even if it be assumed that there was an ambiguity, an excerpt from the Qua Chee Gan decision
would reveal anew the weakness of petitioner's contention. Thus: "Moreover, taking into
account the well known rule that ambiguities or obscurities must be strictly interpreted against
the party that caused them, the 'memo of warranty' invoked by appellant bars the latter from
questioning the existence of the appliances called for in the insured premises, since its initial
expression, 'the undernoted appliances for the extinction of fire being kept on the premises
insured hereby, ... it is hereby warranted ...,' admits of interpretation as an admission of the
existence of such appliances which appellant cannot now contradict, should the parol evidence
rule apply."
7

To the same effect is the following citation from the same leading case: "This rigid application
of the rule on ambiguities has become necessary in view of current business practices. The
courts cannot ignore that nowadays monopolies, cartels and concentration of capital,
endowed with overwhelming economic power, manage to impose upon parties dealing with
them cunningly prepared 'agreements' that the weaker party may not change one whit, his
participation in the 'agreement' being reduced to the alternative to 'take it or leave it' labelled
since Raymond Saleilles 'contracts by adherence' (contrats d'adhesion), in contrast to those
entered into by parties bargaining on an equal footing, such contracts (of which policies of
insurance and international bills of lading are prime examples) obviously call for greater
strictness and vigilance on the part of courts of justice with a view to protecting the weaker
party from abuses and imposition, and prevent their becoming traps for the unwary (New Civil
Code. Article 24; Sent. of Supreme Court of Spain, 13 Dec. 1934, 27 February 1942)."
8

The last error assigned which would find fault with the decision of respondent Court of Appeals
insofar as it affirmed the lower court award for exemplary damages as well as attorney's fees
is, on its face, of no persuasive force at all.
The conclusion that inescapably emerges from the above is the correctness of the decision of
respondent Court of Appeals sought to be reviewed. For, to borrow once again from the
language of the Qua Chee Gan opinion: "The contract of insurance is one of perfect good faith
(uberima fides) not for the insured alone,but equally so for the insurer; in fact, it is more so for
the latter, since its dominant bargaining position carries with it stricter responsibility."
9

This is merely to stress that while the morality of the business world is not the morality of
institutions of rectitude like the pulpit and the academe, it cannot descend so low as to be
another name for guile or deception. Moreover, should it happen thus, no court of justice
should allow itself to lend its approval and support.1awphl.nt
We have no choice but to recognize the monetary responsibility of petitioner Fieldmen's
Insurance Co., Inc. It did not succeed in its persistent effort to avoid complying with its
obligation in the lower court and the Court of Appeals. Much less should it find any receptivity
from us for its unwarranted and unjustified plea to escape from its liability.
WHEREFORE, the decision of respondent Court of Appeals of July 20, 1965, is affirmed in its
entirety. Costs against petitioner Fieldmen's Insurance Co., Inc.
Concepcion, C.J., Reyes, J.B.L., Dizon, Makalintal, Zaldivar, Sanchez, Castro and Angeles, JJ.,
concur.
Republic of the Philippines
SUPREME COURT
Manila
EN BANC
G.R. No. L-47593 September 13, 1941
THE INSULAR LIFE ASSURANCE CO., LTD., petitioner,
vs.
SERAFIN D. FELICIANO and ANGEL, FLORENDA, EUGENIO, HERMINIO and LETICIA, all
surnamed FELICIANO, represented by their guardian ad litem SERAFIN D.
FELICIANO, respondents.
Araneta, Zaragosa, Araneta & Bautista for petitioner.
Delfin Joven for respondents.
LAUREL, J.:
One Evaristo Feliciano filed an application for insurance with the herein petitioner upon the
solicitation of one of its agents. Two insurance policies to the aggregate amount of P25,000
were issued to him. Feliciano died on September 29, 1935. The defendant company refused to
pay on the ground that the policies were fraudulently obtained, the insured having given false
answers and statements in the application as well as in the medical report. The present action
was brought to recover on said policies. The lower court rendered judgment in favor of the
plaintiffs. The lower court found that at the time Feliciano filed his application and at the time
he was subjected to physical examination by the medical examiner of the herein petitioner, he
was already suffering from tuberculosis. This fact appears in the negative both in the
application and in the medical report. The lower court, after an exhaustive examination of the
conflicting testimonies, also found that Feliciano was made to sign the application and the
examiner's report in blank, and that afterwards the blank spaces therein were filled in by the
agent and the medical examiner, who made it appear therein that Feliciano was a fit subject
18

for insurance. The lower court also held that neither the insured nor any member of his family
concealed the real state of health of the insured. That as a matter of fact the insured, as well as
the members of his family, told the agent and the medical examiner that the applicant had
been sick and coughing for sometime and that he had also gone three times to the Santol
Sanatarium. On appeal, this finding of facts of the lower court was sustained by the Court of
Appeals. This concludes the controversy over the facts in so far as this Court is concerned.
The first assignment of error of the petitioner raises the question we are now called upon to
decide:
The Court of Appeals erred in holding that an insurance company has no right to avoid a policy
where its agent knowingly and intentionally wrote down the answers in the application
differing from those made by the insured, in disregard of the exception that when the agent,
instead of serving the interests of his principal, acts in his own or another's interest and
adversely to that of his principal, the said principal is not bound by said acts of the agent."
On the proposition thus presented, there are two main avenues of approach indicated: one
leading to the validation of a policy where its agent, without fraud, collusion or bad faith on
the part of the insured, falsified the answers given by the insured; and the other, leading to the
avoidance of the policy under the circumstances. We see no need for an extended discussion
of the conflicting authorities. Whenever courts are given the choice between two conflicting
principles, the determinative fact which should sway them is the conformity of its
contemplated course to reason and to "the common sense of the situation." The life of the law
is not only logic but experience.
The phenomenal growth of insurance from almost nothing a hundred years ago to its present
gigantic proportion is not of the outstanding marvels of present-day business life. The demand
for economic security, the growing need for social stability, and the clamor for protection
against the hazards of cruel-crippling calamities and sudden economic shocks, have made
insurance one of the felt necessities of modern life. Insurance is no longer a rich man's
monopoly. Upon it are heaped the assured hopes of many families of modest means. It is
woven, as it were, into the very warp and woof of national economy. It touches the holiest and
most sacred ties in the life of man-love of parents, love of wives and love of children. It is of
common knowledge that the selling of insurance today is subjected to the whilrlwind pressure
of modern salesmanship. Insurance companies send detailed instructions to their agents to
solicit and procure applications. These agents are to be found all over the length and breadth
of the land. They are stimulated to more active efforts by contests and by the keen
competition offered by other rival insurance companies. They are supplied with blank
applications and paid large commissions on the policies secured by them. All transactions are
generally done through these agents. They act, in fact and in theory, as the general
representatives of the insurance companies. They supply all the information , prepare and
answer the applications, submit the applications to their companies, conclude the transactions,
and otherwise smooth out all difficulties. The agents, in short, do what the company set them
to do.
In the present case, the agent knew all the time the true state of health of the insured. The
insurer's medical examiner approve the application knowing full well that the applicant was
sick. The situation is one in which one of two innocent parties must bear a loss for his reliance
upon a third person. In this case, it was the insurer who gave the agent authority to deal with
the applicant. It was the one who selected the agent, thus implying that the insured could put
his trust on him. It was the one who drafted and accepted the policy and consummated the
contract. It seems reasonable that as between the two of them, the one who employed and
gave character to the third person as its agent should be the one to bear the loss.
The company received the money of the applicant as the price of the risk to be taken by it. If
the policy should be avoided, it must be because it was void from the very beginning, and the
result would be that the insurer, while it received the money, never assumed any risk. The
result would be, in the language of one of the cases, "to place every simple or uneducated
person seeking insurance at the mercy of the insurer who could, through its agent, insert in
every application, unknown to the applicant and over his signature, some false statements
which would enable him to avoid all liability while retaining the price paid for the supposes
insurance." (State Insurance Company v. Taylor, 14 Colo. 499, 24 Pac. 333.) The weight of
authority is that if an agent of the insurer, after obtaining from an applicant for insurance a
correct and truthful answer to interrogatories contained in the application for insurance,
without knowledge of the applicant fills in false answers, either fraudulently or otherwise, the
insurer cannot assert the falsity of such answers as a defense to liability on the policy, and this
is true generally without regard to the subject matter of the answers or the nature of the
agent's duties or limitations on his authority, at least if not brought to the attention of the
applicant.
The fact that the insured did not read the application which he signed, is not indicative of bad
faith. It has been held that it is not negligence for the insured to sign an application without
first reading it if the insurer by its conduct in appointing the agent influenced the insured to
place trust and confidence in the agent. (Den Hartog v. Home Nat. Ins. Asso., 197 Iowa, 143
196 N. W. 944.) As the court said in the case of Germania L. Ins. Co. v. Lunkebiemer, 127 Ind.
538, 26 N. E. 1082, "Nor can it be said that the assured, who has fully, frankly, truthfully, and in
good faith answered all the required questions, is guilty of negligence in signing, without
reading, the application which is thereupon prepared by the agent. He is justified in assuming
that the agent has, with equal good faith, truthfully recorded the answers given him. He may
well say to the company: "You accredited this man to me as your representative and I signed
the application thus prepared by him, relying upon the character which you gave him when
you commissioned him to come to me as your agent. If he acted dishonestly in the matter, you
and not I must suffer the consequences.' ..." In the instant case, it has been proved that the
insured could not read English, the language in which the application was written, and that
after the contract was signed, it was kept by his mother. As a consequence, the insured had no
opportunity to read or correct any misstatement therein. (Bill of Exceptions, pp. 60-61.)
We have not been insensible to the appeal that the course we have followed may lead to fraud
and work hardship on insurance companies, for it would be easy for insurance agents and
applicants to insert false answers in their applicants to insert false answers in their applications
for insurance. This means that it is to the particular interest of these companies to exercise
greater care in the selection of their agents and examiners. Their protection is still in their own
hands and which may be achieved by other means. Withal, the attainment of a common good
19

may involve impairment and even sacrifice of beneficial interests of a particular group, but in
life, compromise is inevitable until the hour of doom strikes.
The petition is hereby dismissed and the judgment sought to be reviewed is affirmed with
costs against the petitioner. So ordered.
Abad Santos, Diaz, and Horrilleno, JJ., concur.


Separate Opinions
OZAETA, J., with whom concur AVANCEA, C.J., and MORAN, J., dissenting:
Altho a dissenting opinion is but a voice in the wilderness, we have to write it because the
Constitution so requires.
The material facts are not disputed in this instance, but they are not adequately stated in the
majority opinion, and we apprehend that the significance of those not stated therein may have
been overlooked by the majority of the Court.
This is a suit on two life insurance policies issued by the petitioner (hereinafter referred to as
the Company) to Evaristo Feliciano as of October 1 and November 1, 1934, for P20,000 and
P5,000, respectively. The application for the first policy was signed on October 12, and that for
the second policy, on October 28, 1934. On those dates Feliciano "had an advanced disease of
the lungs ... He was breathless, having difficulty in breathing, and he had the appearance of
one with high fever." As a matter of fact, on October 12, 1934, the very day the insured signed
the first applications, after the last X-ray examination of his lungs had been made at the Santol
Sanatorium by Doctor Trepp, the latter informed the respondent Serafin D. Feliciano, brother
of the insured, of the result of the X-ray examination and told him that in his opinion his
brother "was already in a very serious and practically hopeless condition." (Trial court's
decision, P.27, B. of E.) After the first application for insurance of P20,000 had been approved
and the corresponding policy issued, the insured applied on October 28, 1934, for another
insurance of P5,000, and the policy therefor was issued as of November 1, 1934. Less than one
year later, to wit, on September 29, 1935, the insured died of the same malady he had been
suffering-pulmonary tuberculosis.
The Court of Appeals found in effect that the Company's soliciting agent Romulo M. David, in
conclusion with the medical examiner Dr. Gregorio Valdez, knowingly wrote false answers to
the question contained in the applications and in the medical examiner's reports which they
had made the applicant sign in blank in order to secure the Company's approval thereof and
have the corresponding policies credited to the agent in connection with the interprovincial
contest which the Company was then holding among its soliciting agents to boost the sales of
its policies. The Court of Appeals intimates that Agent David bribed Medical Examiner Valdez
with money which the former borrowed from the applicant's mother by way of advanced
payment on the premium. In this connection, it may be mentioned that the premium paid on
the first policy was P1,111.20, and that on the second policy, P277.80, or a total of P1,389,
which the Company offers to refund.
The Court of Appeals also found that before the insured signed the first application and
medical examiner's report, he and the members of his family told the agent and the medical
examiner that he had been sick and coughing for some time and that he had gone three times
to the Santol Sanatorium and had X-ray pictures of his lungs taken; but that in spite of such
information the agent and the medical examiner told them that the applicant was a fit subject
for insurance.
Each of the policies sued upon contains the following stipulations:
This policy and the application therefor constitute the entire contract between the
parties hereto. ... Only the President, or the Manager, acting jointly with the
Secretary of Assistant Secretary (and then only in writing signed by them) have
power in behalf of the Company to issue permits or to modify this or any contract, or
to extend the time for making any premium payment, and the Company shall not be
bound by any promise or representation heretofore or hereafter given by any person
other than the above-named officials, and by them only in writing and signed
conjointly as stated.
The application referred to in and made a part of the policy contains, among others, the
following statements:
18. I [the applicant] hereby declare that all the above statements and answers as
well as those that I may make to the Company's Medical Examiner in continuation of
this application, to be complete, true and correct to the best of my knowledge and
belief, and I hereby agree as follows:
1. That this declaration, with the answers to be given by me to the Medical Examiner,
shall be the basis of the policy and from part of same.
xxx xxx xxx
3. That the said policy shall not take effect until the first premium has been paid and
the policy has been delivered to and accepted by me, while I am in good health.
4. That the agent taking this application has no authority to make, modify or
discharge contracts, or to waive any of the company's rights or requirements.
xxx xxx xxx
20

Upon the facts above set forth, we are of the opinion that respondents are not entitled to
recover the amounts of the policies in question but only the premiums paid thereon, for the
following reasons:
1. Under the very terms of the policies sued upon there is no valid contract of insurance here.
The policies were issued on the basis of the statement subscribed to by the applicant to the
effect that he was and had been in good health. The basis being false, there was no real
meeting of the minds of the parties. The agents had no authority to bind the company thru oral
representations, and less so when such representations were false and fraudulent.
2. The insured and the members of his family who are the respondents herein were not
entirely innocent of bad faith. They were not candid, unsophisticated rustics. They were well to
do and well educated. They were not ignorant of the practices in the life insurance business. In
1924, the insured had taken an insurance policy of P10,000 from the Sun Life Insurance
Company, which, however, he allowed to lapse. The insured was a "proprietor and
agriculturist" (see policy Exhibit E). The respondent Serafin D. Feliciano, brother of the insured,
is a physician who for some years had worked in the Santol Sanatorium with Doctor Trepp
(Exhibit B, p. 16). The most charitable view that one could take of the insured's part in the
transaction is that he, with the approval of his relatives, particularly his mother who furnished
the money with which to pay the premiums and who was named beneficiary to the extent of
P12,000, allowed himself to be used as instrument in the wrongful issuance of the policies in
question by the Company to defraud the latter. It is difficult to believe that in so doing he and
his relatives were not actuated by the desire for lucre. They knew that a person in bad health
let alone one who was "in a very serious and practically hopeless condition" was not
insurable. So they must also have known, or at least they had good reason to suspect, that
Agent David and Medical Examiner Valdez were not acting in good faith when they made the
applicant sign the application in blank and told him (the hopelessly sick man) that he was fit of
insurance. If the applicant and his relatives were acting in good faith, they would have been
curious enough to scrutinize the application and the medical examiner's report contained in
the first policy upon receipt of it, to see whether the medical examiner had correctly stated
therein the state of the applicant's health. It is significant that shortly after they had received
the first policy of P20,000, the insured applied for and secured another policy of P5,000. As
held by the Supreme Court of the United States in the analogous case on New York Life
Insurance company v. Fletcher, 117 U. S. 519: "He could not hold the policy without approving
the action of the agents and thus becoming a participant in the fraud committed. The retention
of the policy was an approval of the application and of its statements. The consequences of
that approval cannot after his death be avoided."
3. Life insurance is a savings institution; it is not a gambling scheme. The premiums paid by the
insured, plus a participation in the profits realized by the life insurance company from the
investment of those premiums, are returned to him if he survives the policy. If, contrary to the
life expectancy of the insured, he dies before the policy matures, the full amount of the
insurance is paid to his beneficiary. The insured is not expected to lose, but neither is he
supposed to expect a windfall or an inordinate gain. That is elemental in every sound business.
The life insurance business is a co-operative enterprise in the sense that the policy-holders as
well as the company are interested in making profits and in avoiding unnecessary or bad
losses. The company is, to a certain extent, a trustee of the funds paid to it by its policyholders.
No insurance company which would issue policies indiscriminately could expect to survive or,
for that matter, be licensed by the Government to do business. That is fundamental. Every
fraud perpetrated upon the company affects the policy-holders because their share in the
profits is thereby unduly minimized. That is why the Government, thru the Insurance
Commissioner, closely supervises the insurance business (see section 169 et seq., The
Insurance Act). We think it is bad law to hold valid a policy procured thru fraud on the life of a
person who was almost on the brink of his grave. Avaricious persons, with the connivance of
unscrupulous agents of insurance companies, could make money on the lives of their relatives
who were expected to die soon, by fraudulently insuring them, and could get away with it, as
in the instant case. The real or ultimate victim is not the company alone but also its numerous
policyholders who have put their savings in it.
It is suggested that the remedy is for the insurance companies to exercise greater care in the
selection of their agents and examiners. As a matter of fact, under the law no one may act as
soliciting agent of an insurance company without authority or license from the Insurance
Commissioner(section 189, The Insurance Act); and the Insurance Commissioner makes a
careful, confidential investigation of the conduct and reputation of the applicant for such
license before issuing the same. But no amount of care taken by both the company and the
Insurance Commissioner in the selection of soliciting agents and medical examiners can insure
the company against bad faithly and the cupidity of the evil-minded. The company would have
to exact a huge bond of every one of its numerous agents and medical examiners to guarantee
his fidelity, and that would be too expensive to make the insurance business profitable. In
other words, the suggested remedy is, we believe, impracticable. The only safe and sound
policy is, not to condone but to condemn fraud under any and all circumstances.
4. If we are to be guided and persuaded by cases adjudicated in other jurisdictions as the Court
of Appeals was in deciding this case, we should follow that decided by the Supreme Court of
the United States upon facts similar or analogous to those obtaining in the instant case, instead
of adopting doctrines laid down by the state supreme courts and inferior federal courts in
cases the facts of which bear little or no anology to those of the case at bar. The case squarely
in point, but which the Court of Appeals rejected, is New York Life Insurance Company v.
Fletcher, supra. In that case it was stated in the application for insurance that the applicant
never had a disease of the kidneys or any serious disease, and had never been seriously ill, and
had no regular medical attendant, whereas he had been afflicted with diabetes, which is a
serious disease of the kidneys, and had been under medical treatment for it, and he actually
died of that disease. The plaintiff therein, however, alleged, and adduced evidence to show:
. . . That two agents of the company at St. Louis, who were personally acquainted with the
assured and knew his past and then physical condition, had solicited him on different occasions
to take out a policy in the company; that he told each of them on those occasions that he did
not believe he was insurable; that they knew he had been in bad health and had been under
medical treatment for diabetes, though he thought he was then well; that they assured him
that he was insurable, that the fact that he had the disease made no difference, and that if he
would take out a policy and pay the premiums required he would have no trouble; that finally,
about the 18th of December, 1877, he consented to take a policy; that they then told him it
would be necessary for him to answer certain questions as a matter of form; that one of them
thereupon read to him certain questions from a printed blank, and as he answered them the
21

other pretended to take down and write in the blank the substance of the answers as given,
not reading over to the assured what he had written, nor consulting him about it, nor
informing him what it was, but saying that what he did was a mere formality; that when he was
asked with respect to his having any disease of the kidneys he replied that his condition was
well known to the agents, who were aware that he had been sick and under treatment by
Doctor Brokaw for diabetes, and that the doctor's office was opposite, and they could go there
and find out everything they wanted to know; that the assured had faithfully answered all the
questions, but the agents inserted in the blanks false answers; that he had no reason to
suppose that the answers were taken down differently from those given; that after answering
all their question he was asked to sign his name to the paper to identify him as the party for
whose benefit the policy was to be issued, and for that purpose he signed the paper twice,
without reading it or the written answers; that the agents did not read to him any part of the
application except the questions, and did not read the clause set forth in the defendant's
answer, nor call the attention to the fact that his signatures were intended as an acceptance or
assent to that clause; that when the policy was delivered to him he neither read it nor the copy
of the application attached to it, that the agent who delivered it informed him that it was right,
and he was insured, and he gave no further attention to the matter; that the annual premiums,
as they fell due, were paid to said agent, who received them with full knowledge of all the
facts; and that, therefore, the company was estopped form pretending that any of the answers
as written rendered the policy void." (117 U.S. 521-523.)
In reversing the judgment rendered by the trial court in favor of the plaintiff, the Federal
Supreme Court held "that the agent had no authority from the company to falsify the
answers," and that "the assured could acquire no right by virtue of his falsified answers." The
Court further said:
. . . Both he and the company were deceived by the fraudulent conduct of the agent.
The assured was placed in the position of making false representations in order to
secure a valuable contract which, upon a truthful report of his condition, could not
have been obtained. By them the company was imposed upon and induced to enter
into the contract. In such a case, assuming that both parties acted in good faith,
justice would require that the contract be cancelled and the premiums returned. As
the present action is not for such a cancellation, the only recovery which the plaintiff
could properly have upon the facts he asserts, taken in connection with the limitation
upon the powers of the agent, is for the amount of the premiums paid, and to that
only would he be entitled by virtue of the statute of Missouri.
But the case as presented by the record is by no means as favorable to him as we
have assumed. It was his duty to read the application he signed. He knew that upon it
the policy would be issued, if issued at all. It would introduce great uncertainty in all
business transactions, if a party making written proposals for a contract, with
representations to induce its execution, should be allowed to show, after it had been
obtained, that he did not know the contents of his proposals, and to enforce it,
notwithstanding their falsity as to matters essential to its obligation and validity.
Contracts could not be made, or business fairly conducted, if such a rule should
prevail; and there is no reason why it should be applied merely to contract of
insurance. There is nothing in their nature which distinguishes them in this particular
from others. But here the right is asserted to prove not only that the assured did not
make the statements contained in his answers, but that he never read the
application, and to recover upon a contract obtained by representations admitted to
be false, just as though they were true. If he had read even the printed lines of his
application, he would have seen that it stipulated that the rights of the company
could in no respect be affected by his verbal statements, or by those of its agents,
unless the same were reduced to writing and forwarded with his application to the
home office. The company, like any other principal, could limit the authority of its
agents, and thus bind all parties dealing with them with knowledge of the limitation.
It must be presumed that he read the application, and was cognizant of the
limitations thereon expressed.
xxx xxx xxx
There is another view of this case equally fatal to a recovery. Assuming that the
answers of the assured were falsified, as alleged, the fact would be at once disclosed
by the copy of the application, annexed to the policy, to which his attention was
called. He would have discovered by inspection that a fraud had been perpetrated,
not only upon himself but upon the company, and it would have been his duty to
make the fact known to the company. He could not hold the policy without
approving the action of the agents and thus becoming a participant in the fraud
committed. The retention of the policy was an approval of the application and of its
statements. The consequences of that approval cannot after his death be avoided.
. . . No one can claim the benefit of an executory contract fraudulently obtained,
after the discovery of the fraud, without approving and sanctioning it. (117 U. S. 529-
530, 534, 535.)
Our attention has been called to al later case Continental Life Insurance
Company v. Chamberlain, 132 U. S. 304 in which the court held the company liable upon a
policy of insurance of the life of one Richard Stevens issued under the following circumstances,
as stated in the decision:
The application for insurance was taken in Iowa by one Boak, a district agent of the
company in certain named counties of the States, fourteen in number, having written
authority "to prosecute the business of soliciting and procuring applications for life
insurance policies within and throughout said territory."
Among the numerous questions propounded in the application was the following:
"Has the said party [the applicant] any other insurance on his life; if so, where and for
what amounts?" The answer, as it appears in the application, is: "No other." That
answer, as were all the answers to questions propounded to the applicant, was
written by the company's agent, Boak. In reference to the above question and
answer, the latter testified: "I asked him [Stevens] the question if he had any other
insurance, as printed in the application and as we ask every applicant, and he told me
he had certain certificates of membership with certain cooperative societies, and he
enumerated different ones, and said he did not know whether I would consider them
22

insurance or not. I told him emphatically that I did not consider them insurance and
we had considerable conversation about it. He wanted to know my authority for
saying I did not consider them insurance. I gave him my authority give him my
reasons and he agreed with me that these cooperative societies were in no sense
insurance companies, and in that light I answered the question "No" after he had
stated the facts? A. I did. Q. Who wrote the answer in there? A. I did."
xxx xxx xxx
It was admitted on the trial that at the date of Stevens' application he had insurance
in cooperative companies to the amount of $12,000. (132 U. S. 306, 308.)
The court, after quoting the pertinent provision of the statute of Iowa, observed that "by force
of the statute, he was the agent of the company in soliciting and procuring the application. He
could not, by any of his, shake of the character of agent for the company. Nor could the
company by any provision in the application or policy convert him into an agent of the
assured." Referring to the incorrectness propounded by him to the applicant in relation to the
stipulation in the policy that the terms thereof could not be varied except in writing signed by
the president or the secretary of the company, the court said:
. . . The purport of the word "insurance" in the question, "Has the said party any
other insurance on his life?" is not so absolutely certain as, in an action upon the
policy, to preclude proof as to what kind of life insurance the contracting parties had
in mind when that question was answered. Such proof does not necessarily
contradict the written contract. Consequently, the above clause, printed on the back
of the policy, is to be interpreted in the light of the statute and of the understanding
reached between the assured and the company by its agent when the application
was completed, namely, that the particular kind of insurance inquired about did not
include insurance in cooperative societies. In view of the statute and of that
understanding, upon the faith of which the assured made his application, paid the
first premium, and accepted the policy, the company is estopped, by every principle
of justice, from saying that its question embraced insurance in cooperative
associations. The answer of "No other" having been written by its own agent,
invested with authority to solicit and procure applications, to deliver policies, and,
under certain limitations, to receive premiums, should be held as properly
interpreting both the question and the answer as to other insurance. (132 U. S. 311-
312.)
There is no conflict between the two cases. They were decided differently because the facts
were different. Suffice it for us to say that the facts of the instant case are analogous to those
of the Fletcher case and different from those of the Chamberlain case.
We have examined the three cases cited in the majority opinion, from the supreme courts of
Colorado, Iowa, and Indiana, respectively, and we find that the facts of each and everyone of
them bear no analogy to those of the present case.
5. The majority opinion says: "The situation is one in which one of two innocent parties must
bear a loss for his reliance upon a third person." We cannot subscribe to this proposition (1)
because, as we have pointed out above, the insured and his relatives, the herein respondents,
were not innocent of bad faith and (2) because, even if the policies in question should be held
invalid, the respondents would not suffer any loss since the Company has offered to return the
premiums paid, and it could be ordered to make such refund with legal interest. By such
judgment neither party would be permitted to enrich himself at the expense of the other. This,
we feel, is urged by justice, reason, and "the common sense of the situation."
Republic of the Philippines
SUPREME COURT
Manila
EN BANC
G.R. No. L-47593 December 29, 1943
THE INSULAR LIFE ASSURANCE CO., LTD., petitioner,
vs.
SERAFIN D. FELICIANO ET AL., respondents.
Manuel Roxas and Araneta, Zaragoza, Araneta and Bautista for petitioner.
Deflfin Joven and Pablo Lorenzo for respondents.
Ramirez and Ortigas as amici curiae.

OZAETA, J.:
In a four-to-three decision promulgated on September 13, 1941,
1
this Court affirmed the
judgment of the Court of Appeals in favor of the respondents and against the petitioner for the
sum of P25,000, representing the value of two insurance policies issued by the petitioner on
the life of Evaristo Feliciano. A motion to reconsider and set aside said decision has been filed
by the petitioner, and both parties have submitted exhaustive and luminous written arguments
in support of their respective contentions.
The facts of the case are set forth in the majority and dissenting opinions heretofore handed
down by this Court, the salient points of which may be briefly restated as follows:
Evaristo Feliciano, who died on September 29, 1935, was suffering with advanced pulmonary
tuberculosis when he signed his applications for insurance with the petitioner on October 12,
1934. On that same date Doctor Trepp, who had taken X-ray pictures of his lungs, informed the
respondent Dr. Serafin D. Feliciano, brother of Evaristo, that the latter "was already in a very
serious ad practically hopeless condition." Nevertheless the question contained in the
application "Have you ever suffered from any ailment or disease of the lungs, pleurisy,
pneumonia or asthma?" appears to have been answered , "No" And above the signature of
23

the applicant, following the answers to the various questions propounded to him, is the
following printed statement:1awphil.net
I declare on behalf of myself and of any person who shall have or claim any interest
in any policy issued hereunder, that each of the above answers is full, complete and
true, and that to the best of my knowledge and belief I am a proper subject for life
insurance. (Exhibit K.)
The false answer above referred to, as well as the others, was written by the Company's
soliciting agent Romulo M. David, in collusion with the medical examiner Dr. Gregorio Valdez,
for the purpose of securing the Company's approval of the application so that the policy to be
issued thereon might be credited to said agent in connection with the inter-provincial contest
which the Company was then holding among its soliciting agents to boost the sales of its
policies. Agent David bribed Medical Examiner Valdez with money which the former borrowed
from the applicant's mother by way of advanced payment on the premium, according to the
finding of the Court of Appeals. Said court also found that before the insured signed the
application he, as well as the members of his family, told the agent and the medical examiner
that he had been sick and coughing for some time and that he had gone three times to the
Santol Sanatorium and had X-ray pictures of his lungs taken; but that in spite of such
information the agent and the medical examiner told them that the applicant was a fit subject
for insurance.
Each of the policies sued upon contains the following stipulations:
This policy and the application herefor constitute the entire contract between the
parties hereto. . . . Only the President, or the Manager, acting jointly with the
Secretary or Assistant Secretary (and then only in writing signed by them) have
power in behalf of the Company to issue permits, or to modify this or any contract,
or to extend the same time for making any premium payment, and the Company
shall not be bound by any promise or representation heretofore or hereafter given by
any person other than the above-named officials, and by them only in writing and
signed conjointly as stated.
The application contains, among others, the following statements:
18. I [the applicant] hereby declare that all the above statements and answers as
well as all those that I may make to the Company's Medical Examiner in continuation
of this application, to be complete, true and correct to the best of my knowledge and
belief, and I hereby agree as follows:
1. That his declaration, with the answers to be given by me to the Medical
Examiner, shall be the basis of the policy and form part of same.
x x x x x x x x x
3. That the said policy shall not take effect until the first premium has been paid and
the policy has been delivered to and accepted by me, while I am in good health.
4. That the agent taking this application has no authority to make, modify or
discharge contracts, or to waive any of the Company's rights or requirements.
5. My acceptance of any policy issued on this application will constitute a ratification
by me of any corrections in or additions to this application made by the Company in
the space provided "For Home Office Corrections or Additions Only." I agree that
photographic copy of this applications as corrected or added to shall constitute
sufficient notice to me of the changes made. (Emphasis added.)
The petitioner insists that upon the facts of the case the policies in question are null and void
ab initio and that all that the respondents are entitled to is the refund of the premiums paid
thereon. After a careful re-examination of the facts and the law, we are persuaded that
petitioner's contention is correct. To the reasons adduced in the dissenting opinion heretofore
published, we only desire to add the following considerations:
When Evaristo Feliciano, the applicant for insurance, signed the application in blank and
authorized the soliciting agent and/or medical examiner of the Company to write the answers
for him, he made them his own agents for that purpose, and he was responsible for their acts
in that connection. If they falsified the answers for him, he could not evade the responsibility
for he falsification. He was not supposed to sign the application in blank. He knew that the
answers to the questions therein contained would be "the basis of the policy," and for that
every reason he was required with his signature to vouch for truth thereof.
Moreover, from the facts of the case we cannot escape the conclusion that the insured acted
in connivance with the soliciting agent and the medical examiner of the Company in accepting
the policies in question. Above the signature of the applicant is the printed statement or
representation: " . . . I am a proper subject for life insurance." In another sheet of the same
application and above another signature of the applicant was also printed this statement:
"That the said policy shall not take effect until he first premium has been paid and the policy as
been delivered to and accepted by me, while I am in good health." When the applicant signed
the application he was "having difficulty in breathing, . . . with a very high fever." He had gone
three times to the Santol Sanatorium and had X-ray pictures taken of his lungs. He therefore
knew that he was not "a proper subject for life insurance." When he accepted the policy, he
knew that he was not in good health. Nevertheless, he not only accepted the first policy of
P20,000 but then and there applied for and later accepted another policy of P5,000.
We cannot bring ourselves to believe that the insured did not take the trouble to read the
answers contained in the photostatic copy of the application attached to and made a part of
the policy before he accepted it and paid the premium thereon. He must have notice that the
answers to the questions therein asked concerning his clinical history were false, and yet he
accepted the first policy and applied for another. In any event, he obligated himself to read the
policy when he subscribed to this statement: "My acceptance of any policy issued on this
application will constitute a ratification by me of any corrections in or additions to this
application made by the Company . . ." By accepting the policy he became charged with
24

knowledge of its contents, whether he actually read it or not. He could not ostrich-like hide his
head from it in order to avoid his part of the bargain and at the same time claim the benefit
thereof. He knew, or was chargeable with knowledge, from the very terms of the two policies
sued upon (one of which is printed in English and the other in Spanish) that the soliciting agent
and the medical examiner had no power to bind the Company by any verbal promise or oral
representation. The insured, therefore, had no right to rely and we cannot believe he relied
in good faith upon the oral representation. The insured, therefore, had no right to rely
and we cannot believe he relied in good faith upon the oral representation of said agent and
medical examiner that he (the applicant) was a fit subject for insurance notwithstanding that
he had been and was still suffering with advanced pulmonary tuberculosis.
From all the facts and circumstances of this case, we are constrained to conclude that the
insured was a coparticipant, and coresponsible with Agent David and Medical Examiner Valdez,
in the fraudulent procurement of the policies in question and that by reason thereof said
policies are void ab initio.
Wheretofore, the motion for reconsideration is sustained and the judgment of the Court of
Appeals is hereby reversed. Let another judgment be entered in favor of the respondents and
against the petitioner for the refund of the premiums amounting to P1,389, with legal interest
thereon from the date of the complaint, and without any finding as to costs.
Moran, Paras and Bocobo, JJ., concur.



Separate Opinions

YULO, C.J., concurring:
I can find no quarrel with the legal considerations and conclusions set forth in the original
decision promulgated by this Court. As general rules of law they find full support not only in
reason and in logic, but also in simple human sense of justice. More so, modern and
complicated practices attendant to the ever growing trade in life insurance demand the
strictest accountability by insurance companies for acts of their authorized agents. In this way
only may the State afford reasonable protection to the unwary public from abuse by such
organizations as may be found to be of questionable moral standards.
But a careful consideration of the evidentiary facts as set forth in the decision of the Court of
Appeals leads me to conclude that the ends of justice would not be serve by the application to
the present case of the rules so enunciated. Rather, to serve the ends of justice the case of the
respondents should be removed from the protection of such rules.
The subject of the insurance policies under consideration is the life of the assured. It is
contended by his beneficiaries that they took these policies on the basis of a life expectancy of
a person gravely stricken with tuberculosis. They have consistently made protestations that
they had so informed the agents of the insurance company. But the policies were issued upon
the life of the assured, as a perfectly normal and healthy person. The error is vital and goes to
the very existence of the contract itself. Who is responsible for the error?
The direct cause, of course, is the false recitals in the application for insurance. While it is true
that it was the agents of the insurance company who filled out such application, yet it was the
assured who, by signing the application in blank, made it possible for the said agents to
procure the issuance of the policies on the basis of false information, in order to suit their own
purposes. Upon the admitted facts, I am of the opinion that in justice and in equity, the
responsibility for the falsifications made by the insurance agents in the preparation of the
insurance application should be laid at the door of the assured and his beneficiaries.
I vote with the majority in granting the motion for reconsideration and in reversing the
decision under review.
HONTIVEROS, J., dissenting:
The reasons given in the dissenting opinion in this case, as published in the Official Gazette of
October 4, 1941 (pp. 2847 to 2855), supplemented by those in the resolution of the majority
on the motion for reconsideration, do not seem to me sufficient to overthrow the decision
rendered by the Court of First Instance, confirmed by the Court of Appeals, and sustained by
this Supreme court in its decision of September 18, 1941. The alleged connivance between the
insured Evaristo Feliciano, the agent Romulo M. David, and the medical examiner Dr. Gregorio
Valdez not only does not clearly appear of record, but on the contrary is denied in the finding
of facts of the courta quo and of the Court of Appeals which cannot be reviewed or altered by
this Court.
The mere fact that the insured signed at the bottom of the application for insurance when
some of its lines intended for answers to certain questions were still in blank, answers which
according to the evidence and to the findings of the two inferior courts he had grounds to
believe will be made in accordance with the information which he and his family had given to
agent David and to Dr. Valdez, does not convert these two persons into agents of the insured
in a way as to make the latter responsible for the acts of the former. That the photostatic
copies of said forms which are attached to the policies object of this case are almost illegible, is
a fact which should be taken into account, together with the other fact that Evaristo Feliciano
does not know English, the language in which those documents are written. In support of this
dissenting opinion, the following authorities may be cited:
The mere failure of the insured to inform himself of the insertion of false answers in
the application which has been filled out by the agent of the insurer does not convict
25

him of lack of good faith. (Vol. 5, Cooley's Briefs on Insurance, 2nd Ed., p. 4136, and
many cases cited.)
The insured is not chargeable with such negligence as will render him liable for false
answers inserted by the agent merely because he signed the application in blank and
trusted the agent to fill out by the agent, without reading it. (Id., p. 4136, and many
cases cited.)
An illiterate person or one who does not understand the English language (as is the
case with Evaristo Feliciano) is not guilty of inexcusable negligence in failing to read
the application or having it read to him, nor can it be said that such person
deliberately made a false statement because he did not read over the application.
(81 ALR 865, 866, W. 117 ALR 796.)
Nor can it be said that the assured, who has fully, frankly, truthfully, and in good faith
answered all the required questions, is guilty of negligence in signing, without
reading, the application which is thereupon prepared by the agent. He is justified in
assuming that the agent, has, with equal good faith, truthfully recorded the answers
give. He may well say to the Company: 'You accredited this man to me as your
representative, and I signed the application thus prepared by him, relying upon the
character which you gave him, when you commissioned him to come to me as your
agent. If he acted dishonestly in the matter, you, and not I, must suffer the
consequences . . .! (Germania Life Ins. Co. vs. Lunkeheimer [1931] Ind., 538; 26 N. E.,
1052)
In such case the acceptance of the policy, with this application attached, does not
require the insured to institute an investigation into its provisions, or the conditions
upon which is was issued, to ascertain whether the agent has acted in good faith,
since, under such circumstances, the insured may rely upon the presumption that he
has been honestly dealt with the insurer. (Otto vs. Hartford Ins. Co., 38 Minn., 423).
Besides, the principles that the insured is not bound to know the contents of the
application, and may rely on the agent's assurances that his answers have been
correctly written will, of course, apply with special force where the insured is
illiterate and unable to read, or is ignorant of the language. (Vol. 5, Cooley's Briefs on
Insurance, 2nd Ed. p. 4138, cases cited.)
And also where the photostatic copies of the application embodied in the policy are
practically illegible, the insured is not bound to know the contents of the application.
(New York Ins. Co. vs. Holpem D.C. 57 Fed. 2nd, 200).
According to the great weight of authority, if an agent of the insurer, after obtaining
from an applicant for insurance a correct and truthful answer to interrogations
contained in the application for insurance, without knowledge of the applicant fills in
false answers, either fraudulently or otherwise, the insurer cannot assert the falsity
of such answers as a defense to the liability on the policy and this is generally
without regard to the subject matter of the answers or the nature of the agent's
duties or limitations on his authority, at least if not brought to the attention of the
applicant. It is equally well settled that if a correct representation is made in a
written application, or the insurance agent issuing the policy is appraised of the true
facts concerning the matter in question, as for instance the title to the insured
premises, but the agent inserts an incorrect statement in the policy, the insurer
cannot rely upon the error in avoidance of its liability". Home Ins. Co. vs. Mendenhall,
154 Ill., 452, 45 NE., 1078, 36 LRA., 374; Phoenix Ins. Co. vs. Tucker, 92 Ill., 64, 34 Am
Rep., 106; Commercial Ins. Co. vs. Spanknoble, 52 Ill., 53, 4 Am. Report, 582; Young
vs. Hartford F. Ins. Co. 45 Iowa, 377, 24 Am. Rep., 754; Welsh vs. London Assur. 151
Pa., 607, 25 A, 142, 21 Am St. Rep., 726 (Taken from Am Juris. on Insurance Vol.
29, par. 843).
An insured may be justified in signing an application in blank at the request of the
insurer's agent, who agrees to fill it in from data furnished by the insured or from an
old application. In fact, an insurer cannot urge the falsity of representations
contained in the policy issued, or in the application, where such representations
were inserted therein, either by the company or its agent, after the application was
signed, without the knowledge or consent of the insured, who has made no such
representations. (Couch on Insurance, Vol. 4, par. 842 b.)
I believe that the motion for reconsideration presented in this case should be denied, not only
because of the weighty reasons relied upon in the decision which it attacks, but also because a
dangerous precedent would otherwise be established, for, with the destruction of the
confidence which the public has hitherto reposed in the duly accredited agents of insurance
companies and in their examining physicians, this branch of the economic life of the people will
have to be unfavorably affected.
Imperial, J., dissents.

Republic of the Philippines
SUPREME COURT
Manila
SECOND DIVISION
G.R. No. L-30685 May 30, 1983
NG GAN ZEE, plaintiff-appellee,
vs.
ASIAN CRUSADER LIFE ASSURANCE CORPORATION, defendant-appellant.
Alberto Q. Ubay for plaintiff-appellee.
26

Santiago F. A lidio for defendant-appellant.

ESCOLIN, J.:
This is an appeal from the judgment of the Court of First Instance of Manila, ordering the
appellant Asian-Crusader Life Assurance Corporation to pay the face value of an insurance
policy issued on the life of Kwong Nam the deceased husband of appellee Ng Gan Zee.
Misrepresentation and concealment of material facts in obtaining the policy were pleaded to
avoid the policy. The lower court rejected the appellant's theory and ordered the latter to pay
appellee "the amount of P 20,000.00, with interest at the legal rate from July 24, 1964, the
date of the filing of the complaint, until paid, and the costs. "
The Court of Appeals certified this appeal to Us, as the same involves solely a question of law.
On May 12, 1962, Kwong Nam applied for a 20-year endowment insurance on his life for the
sum of P20,000.00, with his wife, appellee Ng Gan Zee as beneficiary. On the same date,
appellant, upon receipt of the required premium from the insured, approved the application
and issued the corresponding policy. On December 6, 1963, Kwong Nam died of cancer of the
liver with metastasis. All premiums had been religiously paid at the time of his death.
On January 10, 1964, his widow Ng Gan Zee presented a claim in due form to appellant for
payment of the face value of the policy. On the same date, she submitted the required proof of
death of the insured. Appellant denied the claim on the ground that the answers given by the
insured to the questions appealing in his application for life insurance were untrue.
Appellee brought the matter to the attention of the Insurance Commissioner, the Hon.
Francisco Y. Mandamus, and the latter, after conducting an investigation, wrote the appellant
that he had found no material concealment on the part of the insured and that, therefore,
appellee should be paid the full face value of the policy. This opinion of the Insurance
Commissioner notwithstanding, appellant refused to settle its obligation.
Appellant alleged that the insured was guilty of misrepresentation when he answered "No" to
the following question appearing in the application for life insurance-
Has any life insurance company ever refused your application for insurance
or for reinstatement of a lapsed policy or offered you a policy different
from that applied for? If, so, name company and date.
In its brief, appellant rationalized its thesis thus:
... As pointed out in the foregoing summary of the essential facts in this
case, the insured had in January, 1962, applied for reinstatement of his
lapsed life insurance policy with the Insular Life Insurance Co., Ltd, but this
was declined by the insurance company, although later on approved for
reinstatement with a very high premium as a result of his medical
examination. Thus notwithstanding the said insured answered 'No' to the
[above] question propounded to him. ...
1

The lower court found the argument bereft of factual basis; and We quote with approval its
disquisition on the matter-
On the first question there is no evidence that the Insular Life Assurance
Co., Ltd. ever refused any application of Kwong Nam for insurance. Neither
is there any evidence that any other insurance company has refused any
application of Kwong Nam for insurance.
... The evidence shows that the Insular Life Assurance Co., Ltd. approved
Kwong Nam's request for reinstatement and amendment of his lapsed
insurance policy on April 24, 1962 [Exh. L-2 Stipulation of Facts, Sept. 22,
1965). The Court notes from said application for reinstatement and
amendment, Exh. 'L', that the amount applied for was P20,000.00 only and
not for P50,000.00 as it was in the lapsed policy. The amount of the
reinstated and amended policy was also for P20,000.00. It results,
therefore, that when on May 12, 1962 Kwong Nam answered 'No' to the
question whether any life insurance company ever refused his application
for reinstatement of a lapsed policy he did not misrepresent any fact.
... the evidence shows that the application of Kwong Nam with the Insular
Life Assurance Co., Ltd. was for the reinstatement and amendment of his
lapsed insurance policy-Policy No. 369531 -not an application for a 'new
insurance policy. The Insular Life Assurance Co., Ltd. approved the said
application on April 24, 1962. Policy No. 369531 was reinstated for the
amount of P20,000.00 as applied for by Kwong Nam [Exhs. 'L', 'L-l' and 'L-
2']. No new policy was issued by the Insular Life Assurance Co., Ltd. to
Kwong Nam in connection with said application for reinstatement and
amendment. Such being the case, the Court finds that there is no
misrepresentation on this matter.
2

Appellant further maintains that when the insured was examined in connection with his
application for life insurance, he gave the appellant's medical examiner false and misleading
information as to his ailment and previous operation. The alleged false statements given by
Kwong Nam are as follows:
Operated on for a Tumor [mayoma] of the stomach. Claims that Tumor has
been associated with ulcer of stomach. Tumor taken out was hard and of a
hen's egg size. Operation was two [2] years ago in Chinese General Hospital
by Dr. Yap. Now, claims he is completely recovered.
To demonstrate the insured's misrepresentation, appellant directs Our attention to:
27

[1] The report of Dr. Fu Sun Yuan the physician who treated Kwong Nam at the Chinese
General Hospital on May 22, 1960, i.e., about 2 years before he applied for an insurance policy
on May 12, 1962. According to said report, Dr. Fu Sun Yuan had diagnosed the patient's
ailment as 'peptic ulcer' for which, an operation, known as a 'sub-total gastric resection was
performed on the patient by Dr. Pacifico Yap; and
[2] The Surgical Pathology Report of Dr. Elias Pantangco showing that the specimen removed
from the patient's body was 'a portion of the stomach measuring 12 cm. and 19 cm. along the
lesser curvature with a diameter of 15 cm. along the greatest dimension.
On the bases of the above undisputed medical data showing that the insured was operated on
for peptic ulcer", involving the excision of a portion of the stomach, appellant argues that the
insured's statement in his application that a tumor, "hard and of a hen's egg size," was
removed during said operation, constituted material concealment.
The question to be resolved may be propounded thus: Was appellant, because of insured's
aforesaid representation, misled or deceived into entering the contract or in accepting the risk
at the rate of premium agreed upon?
The lower court answered this question in the negative, and We agree.
Section 27 of the Insurance Law [Act 2427] provides:
Sec. 27. Such party a contract of insurance must communicate to the other,
in good faith, all facts within his knowledge which are material to the
contract, and which the other has not the means of ascertaining, and as to
which he makes no warranty.
3

Thus, "concealment exists where the assured had knowledge of a fact material to the risk, and
honesty, good faith, and fair dealing requires that he should communicate it to the assurer, but
he designedly and intentionally withholds the same."
4

It has also been held "that the concealment must, in the absence of inquiries, be not only
material, but fraudulent, or the fact must have been intentionally withheld."
5

Assuming that the aforesaid answer given by the insured is false, as claimed by the appellant.
Sec. 27 of the Insurance Law, above-quoted, nevertheless requires that fraudulent intent on
the part of the insured be established to entitle the insurer to rescind the contract. And as
correctly observed by the lower court, "misrepresentation as a defense of the insurer to avoid
liability is an 'affirmative' defense. The duty to establish such a defense by satisfactory and
convincing evidence rests upon the defendant. The evidence before the Court does not clearly
and satisfactorily establish that defense."
It bears emphasis that Kwong Nam had informed the appellant's medical examiner that the
tumor for which he was operated on was "associated with ulcer of the stomach." In the
absence of evidence that the insured had sufficient medical knowledge as to enable him to
distinguish between "peptic ulcer" and "a tumor", his statement that said tumor was
"associated with ulcer of the stomach, " should be construed as an expression made in good
faith of his belief as to the nature of his ailment and operation. Indeed, such statement must
be presumed to have been made by him without knowledge of its incorrectness and without
any deliberate intent on his part to mislead the appellant.
While it may be conceded that, from the viewpoint of a medical expert, the information
communicated was imperfect, the same was nevertheless sufficient to have induced appellant
to make further inquiries about the ailment and operation of the insured.
Section 32 of Insurance Law [Act No. 24271 provides as follows:
Section 32. The right to information of material facts maybe waived either
by the terms of insurance or by neglect to make inquiries as to such facts
where they are distinctly implied in other facts of which information is
communicated.
It has been held that where, upon the face of the application, a question appears to be not
answered at all or to be imperfectly answered, and the insurers issue a policy without any
further inquiry, they waive the imperfection of the answer and render the omission to answer
more fully immaterial.
6

As aptly noted by the lower court, "if the ailment and operation of Kwong Nam had such an
important bearing on the question of whether the defendant would undertake the insurance
or not, the court cannot understand why the defendant or its medical examiner did not make
any further inquiries on such matters from the Chinese General Hospital or require copies of
the hospital records from the appellant before acting on the application for insurance. The fact
of the matter is that the defendant was too eager to accept the application and receive the
insured's premium. It would be inequitable now to allow the defendant to avoid liability under
the circumstances."
Finding no reversible error committed by the trial court, the judgment appealed from is hereby
affirmed, with costs against appellant Asian-Crusader life Assurance Corporation.
SO ORDERED.
Makasiar (Chairman), Aquino, Concepcion, Jr., Guerrero and De Castro), JJ., concur.
Abad Santos, J., I reserve my vote

Вам также может понравиться